first inventor to file - pass patent...

33
First Inventor to File Effective Date of First-Inventor-to-File Provisions Statutory Framework Definition of Effective Filing Date for the Claimed Invention Prior Filed U.S. Patent Documents (35 U.S.C. 102(a)(2) prior art disclosures) Effectively Filed Date Effectively Filed Date Based on Foreign Filing Date Applying First-Inventor-to-File Provisions to Patent Applications Prior Public Disclosures (AIA 35 U.S.C. 102(a)(1) Exceptions to Prior Art (AIA 35 U.S.C. 102(b)) Grace Period Common Ownership Exception (AIA 35 U.S.C. 102(b)(2)(C)) Patents Awarded to Inventors Inventor's Own Work Exception (AIA 35 U.S.C. 102(b)(1)(A)) Previous Inventor Disclosure Exception (AIA 35 U.S.C. 102(b)(1)(B) and (b)(2)(B)) Subject Matter Obtained Directly or Indirectly from An Inventor (AIA 35 U.S.C. 102(b)(1)(A), (b)(1)(B), (b)(2)(A), and (b)(2)(B)) USPTO Electronic System Indications of First-Inventor-to-File Applications Statement Under 37 CFR 1.55 or 1.78 Interim Copies of Foreign Priority Documents Foreign Filing Date and Translations Foreign Priority Documents Obviousness Miscellaneous Effective Date of First-Inventor-to-File Provisions Question FITF1000: What is the effective date of the first-inventor-to-file provisions? The effective date for the first-inventor-to-file provisions is March 16, 2013. Back to top Statutory Framework Question FITF1051: Does the AIA provide a date in the future where all applications will be considered as AIA (FITF) applications? No, the applicability of the America Invents Act first-inventor-to-file law (“AIA (FITF)”) or pre-America Invents Act first-to-invent (“pre-AIA (FTI)”) law depends on the effective filing date of a claimed invention as defined in 35 U.S.C. 100(i) and not on an arbitrarily selected cut-off date. Question FITF1052: Will the AIA (FITF) law have any effect on provisional applications filed before March 16, 2013? No, the AIA (FITF) law will not affect any application filed before March 16, 2013.

Upload: lamthien

Post on 03-Jan-2019

224 views

Category:

Documents


0 download

TRANSCRIPT

Page 1: First Inventor to File - Pass Patent Barpasspatentbar.com/wp-content/uploads/Quiz-4-First-Inventor-to-File.pdfFirst Inventor to File Effective Date of First-Inventor-to-File Provisions

First Inventor to File Effective Date of First-Inventor-to-File Provisions

Statutory Framework

Definition of Effective Filing Date for the Claimed Invention

Prior Filed U.S. Patent Documents (35 U.S.C. 102(a)(2) prior art disclosures)

Effectively Filed Date

Effectively Filed Date Based on Foreign Filing Date

Applying First-Inventor-to-File Provisions to Patent Applications

Prior Public Disclosures (AIA 35 U.S.C. 102(a)(1)

Exceptions to Prior Art (AIA 35 U.S.C. 102(b))

Grace Period

Common Ownership Exception (AIA 35 U.S.C. 102(b)(2)(C))

Patents Awarded to Inventors

Inventor's Own Work Exception (AIA 35 U.S.C. 102(b)(1)(A))

Previous Inventor Disclosure Exception (AIA 35 U.S.C. 102(b)(1)(B) and (b)(2)(B))

Subject Matter Obtained Directly or Indirectly from An Inventor (AIA 35 U.S.C. 102(b)(1)(A), (b)(1)(B), (b)(2)(A), and (b)(2)(B))

USPTO Electronic System Indications of First-Inventor-to-File Applications

Statement Under 37 CFR 1.55 or 1.78

Interim Copies of Foreign Priority Documents

Foreign Filing Date and Translations

Foreign Priority Documents

Obviousness

Miscellaneous

Effective Date of First-Inventor-to-File Provisions

Question FITF1000: What is the effective date of the first-inventor-to-file provisions?

The effective date for the first-inventor-to-file provisions is March 16, 2013.

Back to top

Statutory Framework

Question FITF1051: Does the AIA provide a date in the future where all applications will be considered as AIA (FITF) applications?

No, the applicability of the America Invents Act first-inventor-to-file law (“AIA (FITF)”) or pre-America Invents Act first-to-invent

(“pre-AIA (FTI)”) law depends on the effective filing date of a claimed invention as defined in 35 U.S.C. 100(i) and not on an

arbitrarily selected cut-off date.

Question FITF1052: Will the AIA (FITF) law have any effect on provisional applications filed before March 16, 2013?

No, the AIA (FITF) law will not affect any application filed before March 16, 2013.

Page 2: First Inventor to File - Pass Patent Barpasspatentbar.com/wp-content/uploads/Quiz-4-First-Inventor-to-File.pdfFirst Inventor to File Effective Date of First-Inventor-to-File Provisions

Question FITF1053: Do foreign patent documents or publications have to be translated to be available as prior art under 35 U.S.C.

102(a)?

Non-English language foreign patent documents or non-patent literature being relied upon in a rejection must be translated if

necessary for the record to be clear as to the facts relied upon to support the rejection. See MPEP § 706.02 (section II)

and Examination Guidelines for Implementing the First Inventor to File Provisions of the Leahy-Smith America Invents Act, 78 Fed.

Reg. 11059, 11063 (Feb. 14, 2013) (response to comment 14).

Question FITF1054: Did pre-AIA 35 U.S.C. 102(b) and 102(e) disappear after the first-inventor-to-file provision became effective on

March 16, 2013? Will all anticipation rejections be made under 35 U.S.C. 102(a) after March 16, 2013?

If an application is a pre-AIA (FTI) application, examiners will continue to make prior art rejections (if appropriate) on the basis of

publications and activities that qualify as prior art under pre-AIA 35 U.S.C. 102(a), 102(b), 102(e), 102(f) and 102(g). By contrast,

if the application is an AIA (FITF) application, examiners will make prior art rejections (if appropriate) on the basis of publications

and activities that qualify as prior art under 35 U.S.C. 102(a)(1) or 102(a)(2).

Question FITF1055: WWill examiners continue to make 35 U.S.C. 102(e) rejections in AIA (FITF) applications?

No, examiners will not make rejections under pre-AIA 35 U.S.C. 102(e) in AIA (FITF) applications. However, 35 U.S.C. 102(a)(2)

is a “counterpart” of pre-AIA 35 U.S.C. 102(e) and provides for rejections on the basis of U.S. patents, U.S. patent application

publications, or WIPO published PCT applications that designate the United States, that contain subject matter that was effectively

filed before the effective filing date of the claims in the application under examination.

Question FITF1056: Are pre-AIA 35 U.S.C. 102(c), (d), (f), and (g) applicable to AIA (FITF) applications?

Pre-AIA 35 U.S.C. 102(c), (d), and (f) are applicable only to pre-AIA (FTI) applications. Pre-AIA 35 U.S.C. 102(g) applies to pre-

AIA (FTI) applications as well as any AIA (FITF) application that contains or ever contained a claimed invention having an

effective filing date that is before March 16, 2013.

Question FITF1057: How should applicant’s admitted prior art be treated under the AIA?

The AIA does not change the current policy discussed in MPEP § 2129 with respect to the use of applicant’s admissions as prior

art. A statement by an applicant in the specification or made during prosecution identifying the work of another as “prior art” is an

admission that can be relied upon for both anticipation and obviousness determinations. See Examination Guidelines for

Implementing the First Inventor to File Provisions of the Leahy-Smith America Invents Act, 78 Fed. Reg. 11059, 11064, 11075

(Feb. 14, 2013) (response to comment 20 and “Admissions”).

Question FITF1058: For Web references, does the USPTO use archive.org (aka, "The Wayback Machine") to get the actual publication

date rather than printout date of the Web reference?

Yes, examiners use “The Wayback Machine” as a source of information to determine when a Web reference was first made

available to the public.

Back to top

Definition of Effective Filing Date for the Claimed Invention

Page 3: First Inventor to File - Pass Patent Barpasspatentbar.com/wp-content/uploads/Quiz-4-First-Inventor-to-File.pdfFirst Inventor to File Effective Date of First-Inventor-to-File Provisions

Question FITF1100: Under the new section 35 U.S.C. 102(a)(1) introduced by the AIA, what does the term “effective filing date” of a

claimed invention mean?

The “effective filing date” of a claimed invention is defined to be the earlier of: (i) the actual filing date of a nonprovisional

application; or (ii) the date of the earliest application to which the nonprovisional application is entitled to a right of foreign priority

or domestic benefit as to such claimed invention. For example, the effective filing date could be the filing date of (i) an earlier-filed

nonprovisional application or international application designating the United States (benefit under 35 U.S.C. 120,121, or 365(c));

(ii) a provisional application (benefit under 35 U.S.C. 119(e)); or (iii) a foreign application (priority under 35 U.S.C. 119(a)-(d) or

365(a) or (b)).

Question FITF1110: What is the difference between the term “effective filing date” (35 U.S.C. 102(b)(1)) and the term “effectively

filed” date (35 U.S.C. 102(d)) as used in the FITF provisions of the AIA?

The “effective filing date” is relevant in the application under examination, and means the earliest date to which the claims under

examination are entitled to priority or benefit under 35 U.S.C. 119, 120, 121, or 365. The “effectively filed” date is relevant in a

U.S. patent, U.S. patent application publication, or WIPO published PCT application being considered as prior art, and means the

earliest foreign or domestic application to which a U.S. patent, U.S. patent application publication, or WIPO published PCT

application claims priority or benefit under 35 U.S.C. 119, 120, 121, or 365 that describes the subject matter being relied upon in a

prior art rejection where the prior art qualifies under 35 U.S.C. 102(a)(2).

Question FITF1120: What is the difference between the term “effective filing date” (35 U.S.C. 102(b)(1)) and the term “effectively

filed” date (35 U.S.C. 102(d)) as used in the FITF provisions of the AIA?

The “effective filing date” is relevant in the application under examination, and means the earliest date to which the claims under

examination are entitled to priority or benefit under 35 U.S.C. 119, 120, 121, or 365. The “effectively filed” date is relevant in a

U.S. patent, U.S. patent application publication, or WIPO published PCT application being considered as prior art, and means the

earliest foreign or domestic application to which a U.S. patent, U.S. patent application publication, or WIPO published PCT

application claims priority or benefit under 35 U.S.C. 119, 120, 121, or 365 that describes the subject matter being relied upon in a

prior art rejection where the prior art qualifies under 35 U.S.C. 102(a)(2).

Question FITF1122: If an application claims priority to/the benefit of multiple applications (foreign or domestic), will the application

have multiple effective filing dates to the claimed subject matter?

The AIA has not changed current practice for determining priority (the effective filing date) for different claims within a claim

set. The effective filing date is determined on a claim-by-claim, and not on an application basis (i.e., a claimed invention, and not an

application, has an effective filing date). Some claims in an application may be supported by a priority document having one filing

date, and other claims in the application may be supported by a different priority document having a different filing date or may not

be supported by any priority/benefit document.

Question FITF1124: For AIA (FITF) applications, how does the addition of new matter into the application impact the effective filing

date?

Page 4: First Inventor to File - Pass Patent Barpasspatentbar.com/wp-content/uploads/Quiz-4-First-Inventor-to-File.pdfFirst Inventor to File Effective Date of First-Inventor-to-File Provisions

For an application filed on or after March 16, 2013 that discloses and claims only subject matter also disclosed in a previously filed

pre-AIA (FTI) application to which the application is entitled to priority or benefit under 35 U.S.C. 119, 120, 121, or 365, an

amendment (other than a preliminary amendment filed on the same day as such application) seeking to add a claim that is directed to

new matter would not convert the application into an AIA (FITF) application. Introduction of new matter is prohibited by 35 U.S.C.

132(a). Thus the application may not actually “contain” a claim to a claimed invention that is directed to new matter. The MPEP

sets forth the following process for treating amendments that are believed to introduce new matter: (1) a new drawing should not be

entered if the examiner discovers that the drawing contains new matter (MPEP § 608.02); and (2) amendments to the written

description or claims involving new matter are ordinarily entered, but the new matter is required to be cancelled from the written

description and the claims directed to the new matter are rejected under 35 U.S.C. 112(a) (MPEP § 608.04). This process in the

MPEP for treating amendments containing new matter, however, is purely an administrative process for handling an amendment

seeking to introduce new matter in violation of 35 U.S.C. 132(a) and resolving disputes between the applicant and an examiner as to

whether a new drawing or amendment to the written description or claims would actually introduce new matter. See Changes to

Implement the First Inventor to File Provisions of the Leahy-Smith America Invents Act, 78 Fed. Reg. 11024, 11043 (Feb. 14, 2013)

(response to comment 24).

Question FITF1130: Is examination under AIA (FITF) or pre-AIA (FTI) law determined by the actual or the effective filing date of the

application?

Whether an application filed on or after March 16, 2013 is an AIA (FITF) or a pre-AIA (FTI) application is determined by the

effective filing date (as defined in 35 U.S.C. 100(i)) of the claimed invention.

Question FITF1133: If an invention was made prior to March 16, 2013 but was not filed until after March 16, 2013, does the invention

date matter under the AIA?

No, the date of invention is irrelevant under the AIA.

Question FITF1135: What is the effective filing date for a nonprovisional application filed on March 1, 2013 claiming priority to a

foreign application filed on March 1, 2012?

The effective filing date of the nonprovisional application is March 1, 2013. The filing date of the foreign priority application

is not the effective filing date for an invention claimed in a pre-AIA (FTI) application, but the foreign priority date may be used to

overcome certain prior art (e.g., pre-AIA 35 U.S.C. 102 (a)).

Question FITF1138: What is the effective filing date of a claimed invention in a nonprovisional application filed on March 17, 2014

claiming priority to a foreign application filed on March 17, 2013?

The effective filing date of a claimed invention in the nonprovisional application is March 17, 2013, only if the claimed invention is

fully supported by the foreign priority application. The filing date of the foreign priority application can be the effective filing date

for an invention claimed in an AIA (FITF) application if the subject matter of the claim is fully supported in the foreign priority

application.

Question FITF1140: How is the effective filing date of a claim determined for purposes of assessing whether the application should be

examined under AIA (FITF) or pre-AIA (FTI) law?

Page 5: First Inventor to File - Pass Patent Barpasspatentbar.com/wp-content/uploads/Quiz-4-First-Inventor-to-File.pdfFirst Inventor to File Effective Date of First-Inventor-to-File Provisions

For an application filed on or after March 16, 2013, the effective filing date of a claimed invention is the earlier of: (1) the actual

filing date of the patent or the application for patent containing the claimed invention; or (2) the filing date of the earliest application

for which the patent or application is entitled, as to such invention, to a right of priority or the benefit of an earlier filing date under

35 U.S.C. 119, 120, 121, or 365. See Examination Guidelines for Implementing the First Inventor to File Provisions of the Leahy-

Smith America Invents Act, 78 Fed. Reg. 11059, 11073 (Feb. 14, 2013). A priority or benefit application must provide support

under 35 U.S.C. 112(a) for a claimed invention for the application to be entitled, as to that claimed invention, to a right of priority or

the benefit of the filing date of that priority or benefit application date under 35 U.S.C. 119, 120, 121, or 365.

Question FITF1143: Is there a time limit for claiming foreign priority or domestic benefit of foreign or domestic priority and perfecting

the claimed for priority?

Yes, any foreign priority or domestic benefit claim must be filed within four months of the actual filing date of the application or

sixteen months of the filing date of the application for which priority or benefit is claimed. (However, this time limit does not apply

to applications filed before November 29, 2000, or to design applications.) An applicant may submit a priority or benefit claim

outside this time period via petition if the delay in submitting the priority or benefit claim was unintentional. Additionally, to perfect

a foreign priority claim, an applicant must provide a certified copy of the foreign application before a patent is granted, and must

provide a certified copy or interim copy of the foreign application within four months of the actual filing date of the application or

within sixteen months from the filing date of the prior foreign application.

Question FITF1145: When is a U.S. patent or a published U.S. or PCT application available as prior art?

A U.S. patent, a published U.S. application, or a published PCT application designating the United States is available as prior art on

the date that the U.S. patent or published application is “effectively filed.” The date that a U.S. patent or U.S. or PCT published

application is “effectively filed” is the earlier of: (i) the actual filing date of the U.S. patent or U.S. or PCT published application; or

(ii) the filing date of the earliest application to which the U.S. patent or U.S. or PCT published application is entitled to claim a right

of foreign priority or domestic benefit, and which describes the subject matter.

Question FITF1150: How does 35 U.S.C. 120 change for AIA (FITF) applications?

The AIA did not change the requirements for claiming the benefit of a prior-filed U.S. application under 35 U.S.C. 120, except that

the prior-filed U.S. application need not satisfy the “best mode” requirement of 35 U.S.C. 112(a).

Back to top

Prior Filed U.S. Patent Documents (35 U.S.C. 102(a)(2) prior art disclosures)

Question FITF1220: Does 35 U.S.C. 102(a)(2) change the definition of a different inventive entity with the language “names another

inventor”? For example, for an application naming A and B as joint inventors, would a U.S. patent, a U.S. patent application publication,

or a WIPO published application naming B alone be available as prior art under 35 U.S.C. 102(a)(2)? Would a U.S. patent, a U.S. patent

application publication, or a WIPO published application naming A, B, and C be available as prior art under 35 U.S.C. 102(a)(2)?

A U.S. patent, a U.S. patent application publication, or a WIPO published application naming B alone would be prior art under 35

U.S.C. 102(a)(2) with respect to an application naming A and B as joint inventors. Nevertheless, 35 U.S.C. 102(b)(2) provides an

exception to 35 U.S.C. 102(a)(2) where the subject matter disclosed in a prior art reference was obtained directly or indirectly from

the inventor or a joint inventor. In this situation, from the naming of B alone on the U.S. patent, a U.S. patent application

Page 6: First Inventor to File - Pass Patent Barpasspatentbar.com/wp-content/uploads/Quiz-4-First-Inventor-to-File.pdfFirst Inventor to File Effective Date of First-Inventor-to-File Provisions

publication, or a WIPO published application the USPTO considers that the subject matter disclosed in the U.S. patent, a U.S. patent

application publication, or a WIPO published application was obtained from a joint inventor (B) and therefore is not prior art. A

U.S. patent, a U.S. patent application publication, or a WIPO published application naming A, B, and C, also would be prior art

under 35 U.S.C. 102(a)(2) unless the applicant shows (by way of an affidavit or declaration under 37 CFR 1.130(a)) that the subject

matter in the U.S. patent, a U.S. patent application publication, or a WIPO published application being relied upon was obtained

from A and/or B. In that case, the exception under 35 U.S.C. 102(b)(2)(A) would apply.

Question FITF1225: Does "PCT application" mean national stage applications under 35 U.S.C. 371 and not all PCT applications?

The term “PCT application” as used in the guidelines and training materials covers all PCT applications regardless of whether the

PCT application enters the national stage in the United States under 35 U.S.C. 371. A PCT application that designates the United

States is not required to enter the national stage in the United States under 35 U.S.C. 371 for the WIPO publication of the PCT

application to have a prior art effect under 35 U.S.C. 102(a)(2).

Question FITF1230: What does “deemed published” in AIA 35 U.S.C. 102(a)(2) mean?

Under 35 U.S.C. 374, the WIPO publication of a PCT application designating the United States shall be “deemed” a publication

under 35 U.S.C. 122(b). The term “deemed published” in AIA 35 U.S.C. 102(a)(2) encompasses WIPO published PCT applications

that are “deemed” a publication under 35 U.S.C. 122(b) by virtue of 35 U.S.C. 374.

Question FITF1240: Can a patent application not yet published, but that will be published at eighteen months from its filing date, be

used as prior art under 35 U.S.C. 102(a)(2)? Will provisional rejections under 35 U.S.C. 102(a)(2) be made under the assumption that

the application will be published?

A provisional rejection under 35 U.S.C. 102(a)(2) is appropriate only if the applications have at least one inventor in common or

have a common assignee. See MPEP 706.02(f)(2) (provides for provisional rejections on the basis of a copending unpublished

application if the applications have at least one inventor in common or have a common assignee).

Question FITF1243: Can a provisional application qualify as prior art under 35 U.S.C. 102(a)(2)?

No, provisional applications cannot qualify as prior art under 35 U.S.C. 102(a)(2) because they are not publicly

available. Provisional applications are not published and are retained in confidence, unless their benefit is claimed in an application

that has been published or patented. However, a U.S. patent, U.S. patent application publication, or WIPO published PCT

application designating the United States that claims the benefit of a provisional application is prior art under 35 U.S.C. 102(a)(2) as

of the filing date of the provisional application with respect to the subject matter described in the provisional application.

Question FITF1245: Does an application need to be published to be prior art under AIA 35 U.S.C. 102(a)(2)? Can a nonprovisional

application that contains a nonpublication request be prior art under AIA 35 U.S.C. 102(a)(2)?

Unpublished applications per se are not prior art under AIA 35 U.S.C. 102(a)(2). An application must be issued as a U.S. patent or

published as a U.S. patent application publication or WIPO published PCT application to be prior art under AIA 35 U.S.C.

102(a)(2).

Page 7: First Inventor to File - Pass Patent Barpasspatentbar.com/wp-content/uploads/Quiz-4-First-Inventor-to-File.pdfFirst Inventor to File Effective Date of First-Inventor-to-File Provisions

Question FITF1250: For a PCT application to qualify as prior art under 35 U.S.C. 102(a)(2), does it have to designate the United

States?

Yes, a PCT application must designate the United States to have a prior art effect under 35 U.S.C. 102(a)(2). However, PCT

applications filed after a change to the PCT Regulations as of January 1, 2004 automatically designate all contracting countries,

including the United States, unless the applicant specifically chooses not to designate a particular contracting country.

Back to top

Effectively Filed Date

Question FITF1310: What is the “effectively filed” date for a non-English PCT application publication being applied as prior art under

35 U.S.C. 102(a)(2)? Are there language requirements for applying a published PCT application as prior art under 35 U.S.C. 102(a)(2)?

As long as the PCT application designates the United States, there is no requirement under the AIA that the PCT application be

published in English in order for the published PCT application to be applied as of its effectively filed date under 35 U.S.C.

102(a)(2). For PCT applications filed on or after January 1, 2004, the filing of a PCT application will automatically constitute the

designation of all contracting countries to the PCT, unless the applicant specifically chooses not to designate a particular contracting

country. See MPEP § 1801. In addition, there is no critical date under the AIA with regard to the international filing date for the

WIPO published PCT application to be available as prior art under 35 U.S.C. 102(a)(2).

The effectively filed date of any subject matter disclosed in a published PCT application that designates the United States is the

earlier of:

1. the actual filing date of the PCT application or

2. the filing date of the earliest application to which the PCT application is entitled to claim a right of foreign priority or domestic benefit which describes the subject matter.

Question FITF1320: Does the date that subject matter is “effectively filed” under 35 U.S.C. 102(a)(2) include the filing date of any

provisional application or foreign application for which priority or benefit is claimed in a U.S. patent, U.S. patent application publication,

or WIPO published PCT application that designated the United States being used as prior art?

Yes, provided that the subject matter being relied upon is described in the provisional application or foreign application for which

priority or a benefit is claimed by the U.S. patent, U.S. patent application publication, or WIPO published PCT application being

used as prior art under 35 U.S.C. 102(a)(2).

Back to top

Effectively Filed Date Based on Foreign Filing Date

Question FITF1410: Must priority have been perfected in order for a U.S. patent or U.S. patent application publication that claims

priority to a foreign application published in a language other than English to be used as prior art under 35 U.S.C. 102(a)(2)?

Provided that the relevant subject matter is described in the priority document, a U.S. patent, U.S. patent application publication, or

WIPO published PCT application is prior art as of its earliest priority date, regardless of whether the applicant perfected the right to

priority in that patent or application. A copy (certified or interim) of any priority application should be in the file of any patent or

patent application publication. However, should the examiner need a copy where it is not present the examiner may consult with the

Scientific and Technical Information Center (STIC) in order to obtain a copy.

Page 8: First Inventor to File - Pass Patent Barpasspatentbar.com/wp-content/uploads/Quiz-4-First-Inventor-to-File.pdfFirst Inventor to File Effective Date of First-Inventor-to-File Provisions

Question FITF1420: Will the examiner be expected to get a translation of a foreign priority document of a U.S. patent or U.S. patent

application publication when using the U.S. patent or U.S. patent application publication as prior art under 35 U.S.C. 102(a)(2)?

Non-English language documents being relied upon in a rejection must be translated if a translation is necessary for the record to be

clear as to the facts relied upon to support the examiner’s rejection. See MPEP § 706.02 (section II) and Examination Guidelines for

Implementing the First Inventor to File Provisions of the Leahy-Smith America Invents Act, 78 Fed. Reg. 11059, 11063 (Feb. 14,

2013) (response to comment 14). This is a case-by-case determination.

Back to top

Applying First-Inventor-to-File Provisions to Patent Applications

Question FITF2100: Do the first-to-invent provisions apply to all applications filed before March 16, 2013?

Yes. The first-to-invent provisions apply to applications filed before March 16, 2013.

Question FITF2104: Are all applications filed on or after March 16, 2013 subject to the first-inventor-to-file provisions in the AIA?

No, the first-inventor-to-file provisions in the AIA apply only to patent applications that contain or contained at any time: (1) a

claim to a claimed invention that has an effective filing date as defined in 35 U.S.C. 100(i) that is on or after March 16, 2013; or (2)

a designation as a continuation, divisional, or continuation-in-part of an application that contains or contained at any time a claim to

a claimed invention with an effective filing date on or after March 16, 2013.

Neither filing a request for continued examination, nor entering the national stage under 35 U.S.C. 371, constitutes filing a new

application. Accordingly, even if a request for continued examination under 37 CFR 1.114 is filed after March 16, 2013, in an

application that was filed before March 16, 2013, the application remains subject to pre-AIA 35 U.S.C. 102 and 103. Similarly, a

PCT application filed under 35 U.S.C. 363 before March 16, 2013 is subject to pre-AIA 35 U.S.C. 102 and 103, regardless of

whether the application enters the national stage under 35 U.S.C. 371 before or after March 16, 2013.

Additionally, if some of the claims of an application have an effective filing date before March 16, 2013, and other claims have an

effective filing date on or after March 16, 2013, then the application will be subject to the first-inventor-to-file provisions. The first-

inventor-to-file provisions apply to an application that contains, or ever contained, a claim to an invention having an effective filing

date on or after March 16, 2013.

Question FITF2108: Is an application that was filed and abandoned before March 16, 2013 but revived after March 16, 2013 subject to

examination under the AIA or pre-AIA law?

The filing date of an application, rather than the revival date of an application, determines whether the application is examined under

AIA (FITF) or pre-AIA (FTI) law. The application was filed before March 16, 2013, and thus remains a pre-AIA (FTI)

application. Here, the revived application is examined under pre-AIA (FTI) law because the application was filed before March 16,

2013. The first-inventor-to-file provisions of the AIA do not apply to applications filed before March 16, 2013.

Question FITF2110: If an applicant filed a nonprovisional patent application before March 16, 2013, but examination begins after

March 16, 2013, will the application be subject to the first-inventor-to-file provisions?

Page 9: First Inventor to File - Pass Patent Barpasspatentbar.com/wp-content/uploads/Quiz-4-First-Inventor-to-File.pdfFirst Inventor to File Effective Date of First-Inventor-to-File Provisions

No. An application filed before March 16, 2013 will not be subject to the first-inventor-to-file provisions. Instead, the application

will be subject to the first-to-invent provisions of the law in effect on March 15, 2013.

Question FITF2112: If an applicant files a nonprovisional application on April 1, 2013, and may properly claim domestic benefit for

all of the claims to a provisional application filed on April 1, 2012, will the application be subject to the first-inventor-to-file provisions?

No. The nonprovisional application will be examined under the first-to-invent provisions because the all of claims in the

nonprovisional are supported by the provisional application filed before March 16, 2013. The provisional application need not have

included claims.

Question FITF2114: If an applicant files a nonprovisional application on April 1, 2013, properly claiming domestic benefit for some of

the claims to a provisional application filed on April 1, 2012, and also adds new claims not supported by the provisional application, will

the application be subject to the first-inventor-to-file provisions?

Yes. The nonprovisional application will be examined under the first-inventor-to-file provisions because the application contains

claims newly added after the first-inventor-to-file provisions became effective on March 16, 2013.

Question FITF2116: If an applicant files a nonprovisional application on or after March 16, 2013, claiming domestic benefit of or

foreign priority to a different application filed before March 16, 2013, but subsequently deletes the domestic benefit or foreign priority

claim during prosecution, will the application switch from being subject to the first-to-invent provisions to the first-inventor-to-file

provisions?

Yes. When the domestic benefit or foreign priority claim is deleted, the nonprovisional application will switch from being subject to

the first-to-invent provisions to the first-inventor-to-file provisions because the claims will have an effective filing date that is on or

after March 16, 2013.

Question FITF2118: If a provisional application is filed on January 1, 2013, and a nonprovisional application claiming the benefit of

the provisional application is filed on April 1, 2013 with two claims, claim 1 being supported by the provisional application and claim 2

not being supported by the provisional application, is claim 1 treated as pre-AIA?

No, if any claim in an application finds support only in an application filed on or after March 16, 2013, then each claim of the

application (claim 1 and claim 2) is subject to the AIA. In addition, if an AIA (FITF) application also contains, or contained at any

time, any claimed invention having an effective filing date that occurs before March 16, 2013, each claim is also subject to pre-AIA

35 U.S.C. 102(g). Thus, if an application contains, or contained at any time, any claimed invention having an effective filing date

that occurs before March 16, 2013, and also contains, or contained at any time, any claimed invention having an effective filing date

that is on or after March 16, 2013, each claim must be patentable under AIA 35 U.S.C. 102 and 103, as well as pre-AIA 35 U.S.C.

102(g), for the applicant to be entitled to a patent. See Examination Guidelines for Implementing the First Inventor To File

Provisions of the Leahy-Smith America Invents Act, 78 Fed. Reg. 11059, 11083-84 (Feb. 14, 2013).

Question FITF2120: If an international application enters the national stage (35 U.S.C. 371) in the United States after March 16, 2013,

and its international filing date is before March 16, 2013, will the application be examined under the first-inventor-to-file provisions?

No. The pre-AIA first-to-invent provisions will apply to the application because the effective filing date of all claims is before

March 16, 2013. The international stage (PCT) and the national stage are the same application and have the same filing date.

Page 10: First Inventor to File - Pass Patent Barpasspatentbar.com/wp-content/uploads/Quiz-4-First-Inventor-to-File.pdfFirst Inventor to File Effective Date of First-Inventor-to-File Provisions

Question FITF2130: Will an application filed after March 16, 2013 be examined under the first-inventor-to-file provisions of the AIA

if it has a parent application filed before March 16, 2013?

An application filed on or after March 16, 2013 that claims priority to or the benefit of an application filed prior to March 16, 2013 is

a pre-AIA application and will be examined under first-to-invent law unless: (1) the application contains or ever contained a claim

to a claimed invention having an effective filing date on or after March 16, 2013; or (2) the application contains or ever contained a

benefit or priority claim to a prior filed application that contains or ever contained a claim to a claimed invention having an effective

filing date on or after March 16, 2013.

Question FITF2132: Is an application filed on or after March 16, 2013 that is a continuation, divisional, or continuation-in-part of a

pre-AIA (FTI) application examined under the first-inventor-to-file provisions of the AIA or the first-to-invent provisions of pre-AIA

law?

A continuation or divisional application of a pre-AIA (FTI) application is also a pre-AIA (FTI) application since the application does

not add subject matter not present in the prior application. By contrast, a continuation-in-part application may be a pre-AIA (FTI)

application or AIA (FITF) application depending upon whether the continuation-in-part application contains or ever contained a

claim to a claimed invention having an effective filing date on or after March 16, 2013.

Question FITF2134: When filing a continuation application, can an applicant choose to apply the AIA (FITF) or pre-AIA (FTI) if the

parent application is a pre-AIA (FTI) application?

An applicant does not have a “choice” of designating an application as AIA or pre-AIA. The applicant is required to provide a

statement if an application filed on or after March 16, 2013, claims priority to or the benefit of an application filed prior to March 16,

2013, and contains or ever contained a claim to a claimed invention having an effective filing date on or after March 16, 2013.

Question FITF2136: If a continuation-in-part application adds subject matter after March 16, 2013, but has support for some claims

prior to March 16, 2013, is the continuation-in-part application treated as an AIA or pre-AIA (FTI) application? Are some claims AIA

and some claims pre-AIA? Is the application examined under both the AIA and pre-AIA law?

The continuation-in-part application is an AIA (FITF) application. If any claim of the application does not have support in an

application filed prior to March 16, 2013, then the application is an AIA (FITF) application, and each claim in the application is

subject to the AIA. There is never a situation in which some claims in a continuation-in-part application are examined under the

AIA and other claims are examined under pre-AIA law. In addition, if an AIA (FITF) application also contains, or contained at any

time, any claimed invention having an effective filing date that occurs before March 16, 2013, each claim is also subject to pre-AIA

35 U.S.C. 102(g). Thus, if an application contains, or contained at any time, any claimed invention having an effective filing date

that occurs before March 16, 2013, and also contains, or contained at any time, any claimed invention having an effective filing date

that is on or after March 16, 2013, each claim must be patentable under AIA 35 U.S.C. 102 and 103, as well as pre-AIA 35 U.S.C.

102(g), for the applicant to be entitled to a patent. See Examination Guidelines for Implementing the First Inventor To File

Provisions of the Leahy-Smith America Invents Act, 78 Fed. Reg. 11059, 11083-84 (Feb. 14, 2013).

Page 11: First Inventor to File - Pass Patent Barpasspatentbar.com/wp-content/uploads/Quiz-4-First-Inventor-to-File.pdfFirst Inventor to File Effective Date of First-Inventor-to-File Provisions

Question FITF2138: If an application is filed as an AIA (FITF) application, but applicant later shows that all claims have support in an

application filed before March 16, 2013 and submits a delayed claim for domestic benefit or foreign priority to the application filed

before March 16, 2013, will that application be changed to a pre-AIA (FTI) application?

Yes, where an application is filed as an AIA (FITF) application but it is later shown that the application is a pre-AIA (FTI)

application, the application will be examined under pre-AIA (FTI) law. The regulations require that any priority or benefit claim be

filed within four months of the actual filing date of the application or sixteen months of the filing date of the application for which

priority or benefit is claimed. (However, this time limit does not apply to design applications, or to any application filed before

November 29, 2000.) The regulations also permit a priority or benefit claim to be submitted outside of this time period via petition

if the delay in submitting the priority or benefit claim was unintentional. Any petition to accept a delayed priority or benefit claim

must be granted by the Office of Petitions. A TC AIA Specialist would then change the status of the AIA indicator if the grant of

such a petition resulted in the status of an application switching from AIA to pre-AIA.

Question FITF2140: Can a child application be an AIA (FITF) application where its parent is a pre-AIA (FTI) application?

Yes, a child application may be a pre-AIA (FTI) application or an AIA (FITF) application depending upon whether the child

application contains or ever contained a claim to a claimed invention having an effective filing date on or after March 16,

2013. Specifically, if the child application contains a claim to a claimed invention having an effective filing date after March 16,

2013, then the child application is an AIA (FITF) application, even if the parent application is a pre-AIA application.

Question FITF2142: In a continuation-in-part application, what if a dependent claim is added with a new feature, and the new feature is

only supported in an application filed on or after March 16, 2013? Is the continuation-in-part application an AIA (FITF) application?

Yes, the continuation-in-part application is an AIA (FITF) application. If any claim of the application does not have support in an

application filed prior to March 16, 2013, then the application is an AIA (FITF) application, and each claim in the application is

subject to the AIA.

Question FITF2144: If an examiner determines that a claim in a continuation application filed after March 16, 2013 is not supported by

the parent specification filed before March 16, 2013, and the applicant cancels that claim, are the remaining claims examined under pre-

AIA regime?

No, if any claim ever presented in the application does not have support in an application filed prior to March 16, 2013, then the

application is an AIA (FITF) application, and each claim in the application is subject to the AIA. If the claims are amended so that

they all find support in the parent application or all claims that do not find support in the parent application are canceled, the

application will remain an AIA (FITF) application.

Question FITF2146: Are U.S. national stage applications (35 U.S.C. 371) with an international filing date before March 16, 2013

always considered pre-AIA?

Yes, under 35 U.S.C. 363, the filing date of a U.S. national stage applications (35 U.S.C. 371) is the filing date of the international

application.

Page 12: First Inventor to File - Pass Patent Barpasspatentbar.com/wp-content/uploads/Quiz-4-First-Inventor-to-File.pdfFirst Inventor to File Effective Date of First-Inventor-to-File Provisions

Question FITF2148: If a provisional application filed before March 16, 2013 fully discloses an invention but does not claim it, and a

nonprovisional application is filed on or after March 16, 2013 and all claims are drawn to the invention disclosed in the provisional

application, will the nonprovisional application be examined under the first-inventor-to-file provisions of the AIA?

No. As long as each claim ever presented in the nonprovisional application is supported under 35 U.S.C. 112(a) by the disclosure of

the prior provisional application filed before March 16, 2013, the nonprovisional application is a pre-AIA (FTI) application,

regardless of whether the provisional application actually contained claims.

Question FITF2150: If there is an application filed in Japan on January 2, 2013 disclosing two embodiments and five claims, but with

claims directed to only one of the embodiments, and a corresponding application is filed in the United States on or after March 16, 2013,

with claims directed to both the first and the second embodiment, is the later filed corresponding U.S. application a pre-AIA (FTI)

application?

Yes, as long as each claim ever presented in the corresponding U.S. application is fully supported under 35 U.S.C. 112(a) by the

disclosure of the prior filed Japan application, the U.S. application is a pre-AIA (FTI) application, regardless of whether the Japan

application actually contained claims to the invention.

Question FITF2152: If a provisional application is filed on January 1, 2013, and a nonprovisional application claiming the benefit of

the provisional application is filed on June 1, 2013 where the claims in the nonprovisional application are not supported word-for-word

in the provisional application, but one of ordinary skill would understand such support to be present in the provisional application, does

the AIA (FITF) apply to the nonprovisional application?

No. As long as each claim ever presented in the nonprovisional application is fully supported under 35 U.S.C. 112(a) by the

disclosure of the provisional application, the nonprovisional application is a pre-AIA (FTI) application.

Question FITF2160: If the claims are amended after March 16, 2103 in an application in which all claims have an effective filing date

prior to March 16, 2103, will the application remain a pre-AIA (FTI) application?

Yes.

Question FITF2170: What happens if an application claims priority to multiple foreign applications, some of which have a filing date

before March 16, 2013, and some of which have a filing date after March 16, 2013?

The effective filing date of the claimed invention is the filing date of the earliest priority application that provides support for the

claimed invention. If any claimed invention in an application filed on or after March 16, 2013 does not have support in a priority or

benefit application filed before March 16, 2013, the application is an AIA (FITF) application.

Question FITF2172: If an application has a filing date on or after March 16, 2013 but includes a foreign priority claim to an application

filed before March 16, 2013, what is the effective filing date of the claimed invention and does the AIA (FITF) or pre-AIA (FTI) law

apply to the application?

If all claims in the application have support in the foreign priority application, the effective filing date is prior to March 16, 2013 and

the application will be examined under pre-AIA (FTI) law. Note that the AIA definition of “effective filing date” (including foreign

priority dates) is used to determine whether an application is a pre-AIA (FTI) application or an AIA (FITF) application. If it is

Page 13: First Inventor to File - Pass Patent Barpasspatentbar.com/wp-content/uploads/Quiz-4-First-Inventor-to-File.pdfFirst Inventor to File Effective Date of First-Inventor-to-File Provisions

determined, however, that an application is a pre-AIA (FTI) application, the AIA definition of “effective filing date” is not used to

determine whether the claims of the application are patentable under pre-AIA 35 U.S.C. 102 and 103.

Question FITF2180: If a reference is prior art under pre-AIA law, but is not prior art under the AIA, can the applicant "game the

system" by refiling the case under AIA?

If an applicant has a pending application being treated under pre-AIA law, the applicant may choose to file another application

claiming the same subject matter, without claiming benefit of the earlier application. This could result in a reference no longer

qualifying as prior art, but it could also expose the application to new prior art references as a result of the later effective filing

date. Such a strategy may also expose the later filed application to issues of double patenting.

Question FITF2190: What is the standard for "support" in a pre-AIA (FTI) application in order to avoid the application being examined

under the AIA (FITF)?

If an application is filed on or after March 16, 2013, but claims domestic benefit of or foreign priority to an application filed before

that date, all claims in the later application must have support in the earlier pre-March 16, 2013 application in accordance with 35

U.S.C. 112(a) in order for the later application to be treated under pre-AIA law.

Back to top

Prior Public Disclosures (AIA 35 U.S.C. 102(a)(1)

Question FITF2210: Under the first-inventor-to-file provisions, does a public use or sale activity have to occur “in this country” to

qualify as prior art?

No. Under the first-inventor-to-file provisions, a prior use or sale activity may occur anywhere in the world and qualify as prior art

under AIA 35 U.S.C. 102(a)(1). If such public use or sale activity were by the inventor, either directly or indirectly, and not more

than one year before the effective filing date of the claimed invention, then the public use or sale activity would fall into a prior art

exception under 35 U.S.C. 102(b)(1)(A).

Question FITF2220: Is subject matter on display in an inventor’s backyard during a neighborhood cookout a “public disclosure” under

AIA 35 U.S.C. 102(a)(1)?

The public use provision of AIA 35 U.S.C. 102(a)(1) has the same substantive scope, with respect to uses by either the inventor or a

third party, as public uses under pre-AIA 35 U.S.C. 102(b) by unrelated third parties or uses by others under pre-AIA 35 U.S.C.

102(a). See Examination Guidelines for Implementing the First Inventor To File Provisions of the Leahy-Smith America Invents

Act, 78 Fed. Reg. 11059, 11075 (Feb. 14, 2013) (“In public use”). An invention is in public use if it is shown to or used by an

individual other than the inventor under no limitation, restriction, or obligation of confidentiality to the inventor. See Clock Spring,

L.P. v. Wrapmaster, Inc., 560 F.3d 1317, 1325 (Fed. Cir. 2009).

Question FITF2225: Many plant breeders grow or sell their seeds outside the United States prior to filing for a plant patent in the

United States. Under AIA 35 U.S.C. 102(a)(1), will seeds grown or sold outside the United States be available as prior art?

Page 14: First Inventor to File - Pass Patent Barpasspatentbar.com/wp-content/uploads/Quiz-4-First-Inventor-to-File.pdfFirst Inventor to File Effective Date of First-Inventor-to-File Provisions

Yes. The public use or sale of the seeds outside the United States may constitute prior art under AIA 35 U.S.C. 102(a)(1), unless an

exception provision in AIA 35 U.S.C. 102(b)(1) applies.

Question FITF2230: What is the effective prior art date of a Japanese patent application publication that was filed on December 12,

2013, and published on May 12, 2015?

A Japanese patent application publication has a prior art effect under AIA 35 U.S.C. 102(a)(1) as of its publication date—May 12,

2015. Prior art under 35 U.S.C. 102(a)(2), which becomes available as the basis for a rejection as of the date that it was “effectively

filed,” is limited to U.S. patents, U.S patent application publications, and published PCT applications which designate the United

States.

Question FITF2240: Is “by another” eliminated from 35 U.S.C 102(a)(1)?

Yes. 35 U.S.C. 102(a)(1) does not contain the phrase “by another.” Under 35 U.S.C. 102(a)(1), there is no requirement that the

prior art relied upon be by others. Thus, any prior art which falls under 35 U.S.C. 102(a)(1) need not be by another to constitute

potentially available prior art. However, disclosures of the subject matter under 35 U.S.C. 102(a)(1) made one year or less before

the effective filing date of the claimed invention by the inventor or a joint inventor or another who obtained the subject matter

directly or indirectly from the inventor or a joint inventor may fall within an exception under 35 U.S.C. 102(b)(1). See Examination

Guidelines for Implementing the First Inventor to File Provisions of the Leahy-Smith America Invents Act, 78 Fed. Reg. 11059,

11075 (Feb. 14, 2013) (“No requirement of ‘by others.’”)

Question FITF2250: Do foreign patent documents or publications need to be in English to be available as prior art under AIA 35

U.S.C. 102(a)(1)?

No, a foreign patent document or publication is prior art under AIA 35 U.S.C. 102(a)(1) as of its publication date, regardless of the

language of publication. With respect to documents that qualify as prior art under AIA 35 U.S.C. 102(a)(1), the practice under AIA

is the same as practice under pre-AIA law; that is, public disclosure documents including patent documents, foreign documents, and

non-patent-literature are considered prior art under pre-AIA 35 U.S.C. 102(a) and 102(b), regardless of the language of

publication. See MPEP 901.05 and Examination Guidelines for Implementing the First Inventor To File Provisions of the Leahy-

Smith America Invents Act, 78 Fed. Reg. 11059, 11063 (Feb. 14, 2013) (response to comment 14).

Question FITF2255: What is the status of the experimental use exception for a public use under the AIA?

Neither the AIA nor its legislative history expressly addresses whether the experimental use exception applies to a public use under

AIA 35 U.S.C. 102(a)(1), or to a use that makes the invention available to the public under the residual clause of AIA 35 U.S.C.

102(a)(1). Because this doctrine arises infrequently before the Office, and is case-specific when it does arise, the Office will

approach this issue when it arises on the facts presented. See Examination Guidelines for Implementing the First Inventor To File

Provisions of the Leahy-Smith America Invents Act, 78 Fed. Reg. 11059, 11063 (Feb. 14, 2013) (response to comment 12).

Question FITF2260: If a sale activity does not reveal or disclose the claimed invention to the public, does the sale activity qualify as

prior art under AIA 35 U.S.C. 102(a)(1)?

Page 15: First Inventor to File - Pass Patent Barpasspatentbar.com/wp-content/uploads/Quiz-4-First-Inventor-to-File.pdfFirst Inventor to File Effective Date of First-Inventor-to-File Provisions

No, to qualify as prior art under AIA 35 U.S.C. 102(a)(1), the sale activity must make the claimed invention available to the

public. See Examination Guidelines for Implementing the First Inventor To File Provisions of the Leahy-Smith America Invents

Act, 78 Fed. Reg. 11059, 11075 (Feb. 14, 2013) (“On sale “).

Question FITF2265: What constitutes a secret sale for purposes of AIA 35 U.S.C. 102(a)(1)?

An activity (such as a sale, offer for sale, or other commercial activity) is secret if it is among individuals with an obligation of

confidentiality to the inventor. See Examination Guidelines for Implementing the First Inventor To File Provisions of the Leahy-Smith

America Invents Act, 78 Fed. Reg. 11059, 11075 (Feb. 14, 2013) (“On sale “).

Question FITF2270: Must a public use or sale be enabled to qualify as prior art under AIA 35 U.S.C. 102(a)(1)?

No, the enablement inquiry is applicable to the question of whether a claimed invention is described in a patent, published patent

application, or printed publication, but is not applicable to the question of whether a claimed invention is “in public use” or “on

sale.” The AIA does not change this principle of pre-AIA case law. SeeExamination Guidelines for Implementing the First Inventor

To File Provisions of the Leahy-Smith America Invents Act, 78 Fed. Reg. 11059, 11063 (Feb. 14, 2013) (response to comment 10).

Question FITF2275: What does the phrase “otherwise available to the public before the effective filing date of the claimed invention”

as used in AIA 35 U.S.C. 102(a)(1) mean?

The phrase “otherwise available to the public” as used in AIA 35 U.S.C. 102(a)(1) is a “catch-all” language focused on whether the

disclosure was “available to the public,” rather than on the means by which the claimed invention became available to the public or

on whether a disclosure constitutes a “printed publication” or falls within another category of prior art as defined in AIA 35 U.S.C.

102(a)(1). The availability of the subject matter to the public may arise in situations such as a student thesis in a university library, a

poster display or other information disseminated at a scientific meeting, subject matter in a laid-open patent application, a document

electronically posted on the Internet, or a commercial transaction that does not constitute a sale under the Uniform Commercial

Code. Even if a document or other disclosure is not a printed publication, or a transaction is not a sale, either may be prior art under

the “otherwise available to the public” provision of AIA 35 U.S.C. 102(a)(1), provided that the claimed invention is made

sufficiently available to the public. See Examination Guidelines for Implementing the First Inventor To File Provisions of the

Leahy-Smith America Invents Act, 78 Fed. Reg. 11059, 11075 (Feb. 14, 2013) (“Otherwise available to the public.”).

Question FITF2280: If a reference states “available online on March 14, 2013,” would the reference qualify as a printed publication

under 35 U.S.C. 102(a)(1)?

Yes, the reference is available as prior art under 35 U.S.C. 102 (a)(1) as of March 14, 2013. There is a catch-all phrase “otherwise

available to the public” in 35 U.S.C. 102 (a)(1) to cover all types of public disclosures not specifically enumerated in the statute.

Back to top

Exceptions to Prior Art (AIA 35 U.S.C. 102(b))

Question FITF2310: Will the applicant raise an exception to prior art under AIA U.S.C. 102(b)?

Generally, the applicant will raise the issue of an exception being applicable via an affidavit or declaration under 37 CFR 1.130

when confronted with a rejection under AIA 35 U.S.C. 102(a)(1) on a grace period disclosure or a rejection under AIA 35 U.S.C.

102(a)(2). However, it is possible that it will be apparent from the record that a grace period disclosure falls within an exception in

Page 16: First Inventor to File - Pass Patent Barpasspatentbar.com/wp-content/uploads/Quiz-4-First-Inventor-to-File.pdfFirst Inventor to File Effective Date of First-Inventor-to-File Provisions

AIA 35 U.S.C. 102(b)(1), in that a grace period publication may name as authors only persons named as the inventor or a joint

inventor, or the specification may indicate that a grace period disclosure is by the inventor or a joint inventor. See Examination

Guidelines for Implementing the First Inventor To File Provisions of the Leahy-Smith America Invents Act, 78 Fed. Reg. 11059,

11076 (Feb. 14, 2013) (“Grace period inventor disclosure exception”).

Question FITF2320: What form of evidence will an applicant need to present to prove an exception under AIA 35 U.S.C. 102(b)?

The Office has adopted 37 CFR 1.130 as a mechanism for an applicant to submit information to establish the facts and evidence

necessary to rely upon the exception provisions in AIA 35 U.S.C. 102(b). The showing that accompanies such an affidavit or

declaration should provide facts, not conclusions, to show that the disclosure should be disqualified as prior art under AIA 35 U.S.C.

102(b).

Question FITF2330: Is there a special or preferred form for an applicant to use in providing evidence in support of an exception under

AIA 35 U.S.C. 102(b)?

No, due to the fact-specific nature of affidavits and declarations, it is impractical for the Office to suggest a “template” acceptable

affidavit or declaration under 37 CFR 1.130. See Examination Guidelines for Implementing the First Inventor To File Provisions of

the Leahy-Smith America Invents Act, 78 Fed. Reg. 11059, 11065 (Feb. 14, 2013) (response to comment 24).

Question FITF2340: Would an affidavit or declaration alone represent sufficient evidence by the inventor to overcome a rejection

under AIA 35 U.S.C. 102(a)(1) or (a)(2)?

An affidavit or declaration under 37 CFR 1.130 is a mechanism for an applicant to provide the facts and evidence necessary to rely

upon the exception provisions in AIA 35 U.S.C. 102(b). The necessary evidence to disqualify a particular disclosure is fact specific

and must be determined on a case-by-case basis. However, the case law on pre-AIA 35 U.S.C. 102(a) and (e) contains examples of

some affidavits or declarations that were found acceptable to disqualify prior art in pre-AIA (FTI) applications. See Examination

Guidelines for Implementing the First Inventor To File Provisions of the Leahy-Smith America Invents Act, 78 Fed. Reg. 11059,

11065 (Feb. 14, 2013) (response to comment 24).

Question FITF2350: For AIA (FITF) applications, can an applicant swear behind a reference?

No, prior invention is not an issue in AIA (FITF) applications. However, an applicant can show that: (1) a disclosure falls under the

exception in AIA 35 U.S.C. 102(b)(1)(A) or 102(b)(2)(A) because the disclosure was made by the inventor or a joint inventor, or by

another who obtained the subject matter disclosed directly or indirectly from the inventor or a joint inventor; or (2) a disclosure falls

under the exception in AIA 35 U.S.C. 102(b)(1)(B) or 102(b)(2)(B) because the subject matter disclosed had been publicly disclosed

by the inventor or a joint inventor before the date that the disclosure of the subject matter on which the rejection is based is effective

as prior art. Note that these exceptions do not apply to a public disclosure more than one year before the effective filing date of the

claimed invention.

Question FITF2360: How are affidavits and declarations filed under 37 CFR 1.131 different from affidavits and declarations filed

under 37 CFR 1.130?

37 CFR 1.131 affidavits/declarations show prior invention to antedate or “swear behind” grace period prior art under the pre-AIA

law. Using a 37 CFR 1.131 affidavit or declaration with the necessary supporting evidence, an applicant may establish prior

Page 17: First Inventor to File - Pass Patent Barpasspatentbar.com/wp-content/uploads/Quiz-4-First-Inventor-to-File.pdfFirst Inventor to File Effective Date of First-Inventor-to-File Provisions

invention to antedate grace period prior art. By contrast, 37 CFR 1.130, which applies to AIA (FITF) applications, is not concerned

with showing prior invention, as prior invention is not relevant under the AIA. 37 CFR 1.130 is a mechanism to show that an

exception under AIA 35 U.S.C. 102(b) is applicable.

Question FITF2370: Are affidavits under 37 CFR 1.131 and 37 CFR 1.132 still allowed after March 16, 2013 in applications filed

before March 16, 2013?

Yes, any application filed before March 16, 2013 is governed by pre-AIA law, and applicants can continue to use

affidavits/declarations to disqualify prior art pursuant to 37 CFR 1.131 and 1.132 in applications filed before March 16,

2013. See Examination Guidelines for Implementing the First Inventor To File Provisions of the Leahy-Smith America Invents Act,

78 Fed. Reg. 11059, 11083 (Feb. 14, 2013) (“Applications filed before March 16, 2013”). Also, see the pre-AIA guidance provided

in MPEP § 715, 716 and 2136.05 for more information regarding affidavits/declarations filed under 37 CFR 1.131 and 1.132. In

addition, many affidavits under 37 CFR 1.132 (e.g., showing unexpected results) would still be applicable to AIA (FITF)

applications.

Question FITF2380: Is there a duty for an applicant to disclose or point out the inventor’s journal articles published before the filing of

the application?

The duty to disclose information pertinent to patentability as set forth in 37 CFR 1.56 remains in effect after the implementation of

the AIA (FITF) provisions. Since this requirement is limited to the “pertinent to patentability” standard, it may not require an

applicant to disclose all prior inventor disclosures before the filing date of a particular application. However, if the applicant wishes

to trigger a 35 U.S.C. 102(b)(1)(B) or 102(b)(2)(B) exception, the applicant may provide the inventor’s prior public disclosure of the

subject matter.

Back to top

Grace Period

Question FITF2410: Is the one year grace period given automatically?

Yes, the grace period exists by operation of 35 U.S.C. 102(b)(1). An applicant is not required to take any action to be entitled to the

grace period of 35 U.S.C. 102(b)(1), but may need to provide evidence or information to show that a grace period disclosure falls

within an exception in 35 U.S.C. 102(b)(1) for such a disclosure to be eliminated as prior art.

Question FITF2415: Does AIA 35 U.S.C. 102(a)(1) eliminate the one-year statutory bar of

pre-AIA 35 U.S.C. 102(b)?

AIA 35 U.S.C. 102(a)(1) and 102(b)(1) operate in tandem to retain the principle of a one-year statutory time bar. A public

disclosure (e.g., publication, public use, offer for sale) more than one year before the effective filing date of a claimed invention is

available as prior art under AIA 35 U.S.C. 102(a)(1) and cannot fall within the exceptions of AIA 35 U.S.C. 102(b)(1). Thus, a

public disclosure (e.g., publication, public use, offer for sale) more than one year before the effective filing date of a claimed

invention acts as a statutory time bar under AIA 35 U.S.C. 102(a)(1).

Question FITF2420: Are the “grace period” exceptions in AIA 35 U.S.C. 102(b)(1) going to be handled by 37 CFR 1.132 affidavits or

declarations and/or attorney remarks?

Page 18: First Inventor to File - Pass Patent Barpasspatentbar.com/wp-content/uploads/Quiz-4-First-Inventor-to-File.pdfFirst Inventor to File Effective Date of First-Inventor-to-File Provisions

Any affidavit or declaration submitted under 37 CFR 1.132 to show attribution in an AIA (FITF) application will simply be treated

as an affidavit or declaration under 37 CFR 1.130(a). Any facts relied upon in the attorney’s remarks must be supported by the

record (the application as filed or in the publication or patent document in question) or in an affidavit or declaration under 37 CFR

1.130.

Question FITF2425: Can an inventor’s grace period disclosure be used to “swear behind” intervening prior art?

No. An applicant cannot show prior invention (i.e., “swearing behind” a reference) to avoid a rejection under 35 U.S.C.

102(a). However, an applicant can show that a disclosure falls under the exception in 35 U.S.C. 102(b)(1)(B) or 102(b)(2)(B)

because the subject matter of the disclosure had been publicly disclosed previously by the inventor or a joint inventor.

Question FITF2430: If the effective filing date of a claimed invention is June 5, 2014, is June, 5, 2013 included in the grace period?

Yes, a disclosure made on June 5, 2013 (but no earlier) falls within the grace period. The exceptions for grace period disclosures in

AIA 35 U.S.C. 102(b)(1) apply to a disclosure “made 1 year or less before the effective filing date of a claimed invention.”

Question FITF2435: If an AIA (FITF) application was filed less than one year after March 16, 2013, does the grace period cover one

year or only up to March 16, 2013?

The grace period can extend prior to March 16, 2013. For example, an AIA (FITF) application with a claim having an effective

filing date of March 18, 2013 has a one-year grace period back to and including March 18, 2012.

Question FITF2440: Does the one-year grace period under AIA 35 U.S.C. 102(b)(1) include weekends and holidays?

Yes, the grace period includes weekends and holidays. But under pre-AIA 35 U.S.C. 102(b), an applicant’s own publication or

activity will not bar a patent if the one-year period in pre-AIA 35 U.S.C. 102(b) expires on a Saturday, Sunday, or Federal holiday

and the application’s U.S. filing date is the next succeeding business day. See MPEP § 2133. The same principle would be

applicable under the AIan applicant’s prior public disclosure would be considered as being within the one-year period in AIA 35

U.S.C. 102(b)(1) if the date that is one year from the applicant’s own publication or activity expires on a Saturday, Sunday, or

Federal holiday and the effective filing date of the claimed invention is the next succeeding business day.

Question FITF2445: Is a disclosure by an inventor during the grace period available as prior art under 35 U.S.C. 102(a)(1) if it does not

disclose every feature of the claimed invention?

Regardless of the subject matter disclosed, an inventor’s disclosure during the grace period is not available as prior art under 35

U.S.C. 102(a)(1).

Question FITF2450: My co-inventor disclosed our invention at a trade show one month before we filed our U.S. nonprovisional

application. Will that disclosure prevent us from obtaining a patent?

No. Regardless of whether the application was filed before or after the AIA first-inventor-to-file provisions took effect on March

16, 2013, a disclosure made one month prior to the filing date of the non-provisional application is not prior art to the claimed

invention by virtue of the one-year grace period available to an inventor.

Page 19: First Inventor to File - Pass Patent Barpasspatentbar.com/wp-content/uploads/Quiz-4-First-Inventor-to-File.pdfFirst Inventor to File Effective Date of First-Inventor-to-File Provisions

Question FITF2455: Is an applicant required to submit his or her prior public disclosure made during the grace period before a first

action on the merits for the applicant to get the benefit of the grace period?

No, an applicant may show a prior public disclosure in an affidavit or declaration under 37 CFR 1.130 in response to a

rejection. However, to facilitate compact prosecution, the applicant may identify a prior grace period public disclosure in the

application upon filing under 37 CFR 1.77.

Question FITF2460: Do unpublished records of the inventions from the inventor establish the grace period?

No, the grace period is the one-year time interval preceding the effective filing date of the claimed invention. An inventor’s

unpublished records of invention could be used to show that a disclosure of subject matter is by another who obtained the subject

matter disclosed directly or indirectly from the inventor or a joint inventor and therefore that the disclosure falls under the exception

in AIA 35 U.S.C. 102(b)(1)(B) or 102(b)(2)(B).

Question FITF2465: If the claimed invention was publicly disclosed (i.e., described, in public use, or on sale) before the filing date of

the invention and outside of the grace period, does the disclosure constitute prior art under 35 U.S.C. 102(a)(1)?

Yes. If there is a public disclosure more than one year before the effective filing date of the claimed invention (i.e., before the grace

period), the disclosure is prior art under 35 U.S.C. 102(a)(1) and none of the exceptions in 35 U.S.C. 102(b) is applicable, regardless

of whether the disclosure is the regardless of whether the disclosure is the inventor’s own work

Question FITF2470: For the inventor's "disclosure" exception under 35 U.S.C. 102(b)(1)(A) or (b)(1)(B), does the term "disclosure"

have the same scope as previously applied in pre-AIA 35 U.S.C. 102 (i.e., publication, on sale, public use)?

A grace period disclosure that establishes that an exception under AIA 35 U.S.C. 102(b)(1)(A) or (b)(1)(B) applies may be in any

form, including publication, public sale, and public use. However, there is no requirement that the grace period disclosure be

enabling.

Back to top

Common Ownership Exception (AIA 35 U.S.C. 102(b)(2)(C))

Question FITF2510: Can “common ownership” statements be used to overcome rejections made under AIA 35 U.S.C. 102(a)(2)?

Yes, a statement of “common ownership” can be used to invoke the exception under AIA 35 U.S.C. 102(b)(2)(C), which states that a

disclosure made in an application or patent shall not be prior art under AIA 35 U.S.C. 102(a)(2) if the subject matter and the claimed

invention were commonly owned or subject to an obligation of assignment to the same person not later than the effective filing date

of the claimed invention. Accordingly, an applicant can disqualify a U.S. or PCT patent document by establishing common

ownership or obligation of assignment by making a clear and conspicuous statement. See Examination Guidelines for Implementing

the First Inventor To File Provisions of the Leahy-Smith America Invents Act, 78 Fed. Reg. 11059, 11079-80 (Feb. 14, 2013)

(“Prior art exception under AIA 35 U.S.C. 102(b)(2)(C) to AIA 35 U.S.C. 102(a)(2) (common ownership or obligation of

assignment)”).

Question FITF2520: With respect to the exception under AIA 35 U.S.C. 102(b)(2)(C), what if the invention under examination

(invention “A”) was commonly owned with invention “B” prior to the effective filing of invention “A” but after the effective filing of

invention “B”?

Page 20: First Inventor to File - Pass Patent Barpasspatentbar.com/wp-content/uploads/Quiz-4-First-Inventor-to-File.pdfFirst Inventor to File Effective Date of First-Inventor-to-File Provisions

The exception in AIA 35 U.S.C. 102(b)(2)(C) would apply to the claimed invention “A,” since the subject matter disclosed and the

claimed invention, not later than the effective filing date of the claimed invention (claimed invention “A”), were owned by the same

person or subject to an obligation of assignment to the same person. The exception in AIA 35 U.S.C. 102(b)(2)(C) would not apply

to the claimed invention “B,” since the common ownership did not exist at the effective filing date of invention “B.” However, the

disclosure of invention “A” would not available as prior art under AIA 35 U.S.C. 102(a)(2) with respect to invention “B.”

Question FITF2530: With respect to the exception under AIA 35 U.S.C. 102(b)(2)(C), does common ownership have to continue after

the filing date of the claimed invention to disqualify prior art under the exception in AIA 35 U.S.C. 102(b)(2)(C)?

Yes, the exception under AIA 35 U.S.C. 102(b)(2)(C) applies only so long as its condition of common ownership (i.e., owned by the

same person or subject to an obligation of assignment to the same person) continues.

Question FITF2540: When the exception in AIA 35 U.S.C. 102(b)(2)(C) is applicable, does it present a double patenting situation?

Yes, if the two applications are claiming patentably indistinct inventions, a possible double patenting situation may exist. The AIA

did not change double patenting practice.

Question FITF2550: Does the exception to prior art under the common ownership provision in 35 U.S.C. 102(b)(2)(C) apply if used for

an obviousness rejection?

Yes. The common ownership exception of 35 U.S.C. 102(b)(2)(C) applies to prior art defined under 35 U.S.C. 102(a)(2), regardless

of whether that art is used in making an anticipation or an obviousness rejection.

Question FITF2560: If a reference and application have the same assignee at the time the application is filed, is that enough to remove

the reference under AIA 35 U.S.C. 102(b)(2)(C), or must common ownership exist at the time of invention?

The exception in AIA 35 U.S.C. 102(b)(2)(C) disqualifies subject matter disclosed in a U.S. patent, U.S. patent application

publication, or WIPO published application from constituting prior art under AIA 35 U.S.C. 102(a)(2) if the subject matter disclosed

and the claimed invention, not later than the effective filing date of the claimed invention, were owned by the same person or subject

to an obligation of assignment to the same person. See Examination Guidelines for Implementing the First Inventor To File

Provisions of the Leahy-Smith America Invents Act, 78 Fed. Reg. 11059, 11079 (Feb. 14, 2013) (Prior art exception under AIA 35

U.S.C. 102(b)(2)(C) to AIA 35 U.S.C. 102(a)(2) (“Common ownership or obligation of assignment)”). The “time of the invention”

is never relevant under AIA 35 U.S.C. 102 or 103. A clear and conspicuous statement by the applicant that the claimed invention of

the application under examination and the subject matter disclosed in the U.S. patent, U.S. patent application publication, or WIPO

published application (prior art) to be excluded under AIA 35 U.S.C. 102(b)(2)(C) were owned by the same person or subject to an

obligation of assignment to the same person not later than the effective filing date of the claimed invention will be sufficient to

establish that the AIA 35 U.S.C. 102(b)(2)(C) exception applies. See Examination Guidelines for Implementing the First Inventor

To File Provisions of the Leahy-Smith America Invents Act, 78 Fed. Reg. 11059, 11080 (Feb. 14, 2013) (Prior art exception under

AIA 35 U.S.C. 102(b)(2)(C) to AIA 35 U.S.C. 102(a)(2) (“Common ownership or obligation of assignment)”).

Question FITF2570: Does AIA 35 U.S.C. 102(b)(2)(C) include a joint research agreement exception similar to the provisions of pre-

AIA 35 U.S.C. 103(c)?

Page 21: First Inventor to File - Pass Patent Barpasspatentbar.com/wp-content/uploads/Quiz-4-First-Inventor-to-File.pdfFirst Inventor to File Effective Date of First-Inventor-to-File Provisions

Yes, AIA 35 U.S.C. 102(c) specifically provides that subject matter disclosed and a claimed invention shall be deemed to have been

owned by the same person or subject to an obligation of assignment to the same person in applying the provisions of AIA 35 U.S.C.

102(b)(2)(C) if:

(1) the subject matter disclosed was developed and the claimed invention was made by, or on behalf of, one or more parties to a joint

research agreement that was in effect on or before the effective filing date of the claimed invention; (2) the claimed invention was

made as a result of activities undertaken within the scope of the joint research agreement; and (3) the application for patent for the

claimed invention discloses or is amended to disclose the names of the parties to the joint research agreement.

Back to top

Patents Awarded to Inventors

Question FITF2600: Once the first-inventor-to-file provisions become effective on March 16, 2013, would someone who copies my

idea and files a patent application on the subject matter before I do be entitled to a patent?

No. In accordance with 35 U.S.C. 101, only inventors are entitled to a patent. Someone who copies another’s idea cannot be an

inventor and therefore will not be entitled to a patent.

Question FITF2610: For applications examined under first-inventor-to-file provisions, how will an examiner address a situation of

improper inventorship since the pre-AIA basis for rejecting under 35 U.S.C. 102(f) does not exist?

In the rare situation where it clear that the application does not name the correct inventorship and the applicant has not filed a request

to correct inventorship under 37 C.F.R. 1.48, the Office will reject the claims under 35 U.S.C. 101 and 35 U.S.C. 115.

Back to top

Inventor's Own Work Exception (AIA 35 U.S.C. 102(b)(1)(A))

Question FITF2720: Does the inventive entity have to be the same for the grace period exception in AIA 35 U.S.C. 102(b)(1)(A) to

apply?

No, an applicant may show that a disclosure falls under the exception in AIA 35 U.S.C. 102(b)(1)(A) because the disclosure was

made by a joint inventor (rather than the inventive entity), and may show that a disclosure falls under the exception in AIA 35

U.S.C. 102(b)(1)(B) because the subject matter disclosed had been publicly disclosed by a joint inventor (rather than the inventive

entity) before the date that the disclosure of the subject matter on which the rejection is based is effective as prior art.

Question FITF2730: What kind of evidence is sufficient to prove the exception of AIA 35 U.S.C. 102(b)(1)(A) to overcome a rejection

under AIA 35 U.S.C. 102(a)(1)?

The Office has adopted 37 CFR 1.130(a) as a mechanism for an applicant to submit information to establish the facts and evidence

necessary to rely upon the exception provisions in AIA 35 U.S.C. 102(b)(1)(A). The showing that accompanies the affidavit or

declaration should provide facts, not conclusions, to show that the disclosure was by the inventor, a joint inventor, or another who

had obtained the subject matter from the inventor or joint inventor. The evidence required under 37 CFR 1.130(a) to disqualify a

publication or patent document under AIA 35 U.S.C. 102(b)(1)(A) or (b)(2)(A) is similar to that required for an affidavit or

declaration of attribution under In re Katz (i.e., an affidavit or declaration of attribution as discussed in MPEP § 716.10). Where the

authorship of the prior art disclosure includes the inventor or a joint inventor named in the application, an “unequivocal” statement

from the inventor or a joint inventor that he/she (or some specific combination of named inventors) invented the subject matter of the

Page 22: First Inventor to File - Pass Patent Barpasspatentbar.com/wp-content/uploads/Quiz-4-First-Inventor-to-File.pdfFirst Inventor to File Effective Date of First-Inventor-to-File Provisions

disclosure, accompanied by a reasonable explanation of the presence of additional authors, may be acceptable in the absence of

evidence to the contrary. However, a mere statement from the inventor or a joint inventor without any accompanying reasonable

explanation may not be sufficient where there is evidence to the contrary. SeeExamination Guidelines for Implementing the First

Inventor To File Provisions of the Leahy-Smith America Invents Act, 78 Fed. Reg. 11059, 11080 (Feb. 14, 2013) (“Showing That

the Disclosure Was Made by the Inventor or a Joint Inventor”). This is similar to the current process for disqualifying a publication

as not being by “others” discussed in MPEP § 2132.01, except that AIA 35 U.S.C. 102(b)(1)(A) requires only that the disclosure be

by the inventor or a joint inventor.

Question FITF2740: Does the exception in 35 U.S.C. 102(b)(1)(A) cover the assignee if the prior art that is available under 35 U.S.C.

102(a)(1) shares the same assignee but not the same inventors?

The exception in 35 U.S.C. 102(b)(1)(A) is limited to disclosures made by the inventor or a joint inventor or by another who

obtained the subject matter disclosed directly or indirectly from the inventor or a joint inventor. The exception in 35 U.S.C.

102(b)(1)(A) would cover a disclosure by an assignee only if the assignee obtained the subject matter disclosed directly or indirectly

from the inventor or a joint inventor. See Examination Guidelines for Implementing the First Inventor to File Provisions of the

Leahy-Smith America Invents Act, 78 Fed. Reg. 11059, 11065 (Feb. 14, 2013) (response to comment 27).

Back to top

Previous Inventor Disclosure Exception (AIA 35 U.S.C. 102(b)(1)(B) and (b)(2)(B))

Question FITF2810: The exception in 35 USC 102(b)(2)(B) applies when the potential prior art “subject matter disclosed” had, before

such disclosure, been “publicly disclosed by the inventor or a joint inventor or another who obtained the subject matter disclosed directly

or indirectly from the inventor or a joint inventor.” What is the difference between "disclosed" and "publicly disclosed?”

The term ‘‘disclosure’’ in 35 U.S.C. 102 may refer to either public or non-public disclosures. Because the exception under 35 USC

102(b)(2)(B) applies specifically to U.S. patents, U.S. patent application publications, or WIPO published PCT applications being

considered as prior art under 35 USC 102(a)(2) as of the effectively filed date (see Question FITF1110 above), the term “disclosure”

in AIA 35 U.S.C. 102(b)(2)(B) refers to a non-public disclosure. U.S. patents, U.S. patent application publications, and WIPO

published PCT applications are disclosures as of their earliest effectively filed date. This is not the date on which the disclosure was

made publicly available. The showing required to establish a public disclosure is discussed in the Examination Guidelines for

Implementing the First Inventor to File Provisions of the Leahy-Smith America Invents Act, 78 Fed. Reg. 11059, 11080-81 (Feb. 14,

2013).

Question FITF2820: What constitutes a sufficient publication under AIA 35 U.S.C. 102(b)(1)(B) or 102(b)(2)(B) for an inventor to

remove an intervening reference? Would an email from an employee to a boss containing a description of the invention qualify as a

“publication” to remove a reference in the grace period?

A publication per se is not required to trigger the exception in either AIA 35 U.S.C. 102(b)(1)(B) or 102(b)(2)(B). Rather, AIA 35

U.S.C. 102(b)(1)(B) and (b)(2)(B) require a previous public disclosure of the subject matter, which could be in the form of a

publication, but could also be in the form of a public use or sale as well. An email from an employee to a boss is not likely to be

considered a public disclosure of the subject matter contained in the email in the absence of evidence that the email was released to

the public.

Page 23: First Inventor to File - Pass Patent Barpasspatentbar.com/wp-content/uploads/Quiz-4-First-Inventor-to-File.pdfFirst Inventor to File Effective Date of First-Inventor-to-File Provisions

Question FITF2830: Would an inventor’s publication have to be enabling and fully supportive of his invention for it to disqualify

another’s publication as prior art under the exception in AIA 35 U.S.C. 102(b)(1)(B) or (b)(2)(B)? Must an inventor’s public use or sale

be enabled to disqualify another’s disclosure as prior art under the exception in AIA 35 U.S.C. 102(b)(1)(B) or (b)(2)(B)?

An affidavit or declaration under 37 CFR 1.130(a) or (b) need not demonstrate that the public disclosure (e.g., publication, use, sale

activity) by the inventor, a joint inventor, or another who obtained the subject matter disclosed directly or indirectly from an inventor

or a joint inventor was an “enabling” disclosure of the subject matter within the meaning of 35 U.S.C. 112(a). An affidavit or

declaration under 37 CFR 1.130(b) must show that: (1) the disclosure in question was made by the inventor or a joint inventor, or

the subject matter disclosed was obtained directly or indirectly from the inventor or a joint inventor (37 CFR 1.130(a)); or (2) the

subject matter disclosed had, before such disclosure was made or before such subject matter was effectively filed, been publicly

disclosed by the inventor or a joint inventor or another who obtained the subject matter disclosed directly or indirectly from the

inventor or a joint inventor (37 CFR 1.130(b)). See Examination Guidelines for Implementing the First Inventor To File Provisions

of the Leahy-Smith America Invents Act, 78 Fed. Reg. 11059, 11081 (Feb. 14, 2013) (“Enablement”). However, where there is an

intervening grace period disclosure of subject matter by another that is fully enabling, a previous non-enabling publication, public

use, or sale activity by the inventor may not be a disclosure of “the subject matter” disclosed in the intervening grace period

disclosure. As such, the intervening disclosure by another would be available as prior art (i.e., the exception in AIA U.S.C.

102(b)(1)(B) or (b)(2)(B) would not apply).

Question FITF2840: For the exception in AIA 35 U.S.C. 102(b)(1)(B) to apply, must the exception in AIA 35 U.S.C. 102(b)(1)(A) also

apply?

No, the exception in AIA 35 U.S.C. 102(b)(1)(B) does not expressly require that the inventor’s prior public disclosure have been

made during the grace period as required for the exception in AIA 35 U.S.C. 102(b)(1)(A). However, if the inventor’s prior public

disclosure is made outside the grace period, the inventor’s prior public disclosure itself is prior art under AIA 35 U.S.C. 102(a)(1)

that is not covered by any exception in AIA 35 U.S.C. 102(b).

Question FITF2850: Under AIA 35 U.S.C. 102(b)(1)(B) and (b)(2)(B), by what date would the inventor need to prove a prior

disclosure of the same subject matter to satisfy the exception?

Under AIA 35 U.S.C. 102(b)(1)(B), the applicant must show a public disclosure by the inventor, a joint inventor, or another who

obtained the subject matter from the inventor before the date the subject matter was patented, published, in public use, on sale, or

otherwise available to the public ” under AIA 35 U.S.C. 102(a)(1). Under AIA 35 U.S.C. 102(b)(2)(B), the applicant must show a

public disclosure by the inventor, a joint inventor, or another who obtained the subject matter from the inventor before the date the

subject matter of the U.S. patent, U.S. patent application publication, or WIPO published application was “effectively filed” under

AIA 35 U.S.C. 102(a)(2).

Question FITF2860: To trigger the exception in AIA 35 U.S.C. 102(b)(1)(B) or (b)(2)(B), should an inventor who is capable of

proving a “prior inventor public disclosure” of “subject matter” disclosed in either an earlier publication or an earlier patent document

submit that evidence in the form of an affidavit/declaration or some other form?

The Office has adopted 37 CFR 1.130(b) as a mechanism for an applicant to submit information to establish the facts and evidence

necessary to rely upon the exception provisions in AIA 35 U.S.C. 102(b)(1)(B) and (b)(2)(B). An inventor seeking to disqualify an

Page 24: First Inventor to File - Pass Patent Barpasspatentbar.com/wp-content/uploads/Quiz-4-First-Inventor-to-File.pdfFirst Inventor to File Effective Date of First-Inventor-to-File Provisions

earlier publication disclosed during the inventor’s “grace period” under the exception outlined in AIA 35 U.S.C. 102(b)(1)(B) could

submit an affidavit or declaration under 37 CFR 1.130(b). An affidavit or declaration under 37 CFR 1.130(b) must identify the

subject matter publicly disclosed and provide the date such subject matter was publicly disclosed by the inventor or a joint inventor

or another who obtained the subject matter disclosed directly or indirectly from the inventor or a joint inventor. In addition, if the

subject matter publicly disclosed on that date was in a printed publication, the affidavit or declaration must be accompanied by a

copy of the printed publication. If the subject matter publicly disclosed on that date was not in a printed publication, the affidavit or

declaration must describe the subject matter with sufficient detail and particularity to determine what subject matter had been

publicly disclosed on that date by the inventor or a joint inventor or another who obtained the subject matter disclosed directly or

indirectly from the inventor or a joint inventor. See Changes To Implement the First Inventor To File Provisions of the Leahy-Smith

America Invents Act, 78 Fed. Reg. 11024, 11058 (Feb. 14, 2013).

Back to top

Subject Matter Obtained Directly or Indirectly from An Inventor (AIA 35 U.S.C. 102(b)(1)(A), (b)(1)(B),

(b)(2)(A), and (b)(2)(B))

Question FITF2910: What does “obtained” mean as used in 35 U.S.C. 102(b)(1) and (b)(2)?

The AIA does not provide a specific definition for the term “obtained.” In context, subject matter disclosed by another must

originate from the inventor or joint inventor in order for the subject matter disclosed to have been obtained, directly or indirectly,

from the inventor or joint inventor as the term is used in 35 U.S.C. 102(b)(1) and (b)(2).

Question FITF2915: What does “directly” or “indirectly” mean as used in AIA 35 U.S.C. 102(b)(1) and (b)(2)? What does

“another…obtained the subject matter directly or indirectly from the inventor or joint inventor” mean as used in AIA 35 U.S.C.

102(b)(1) and (b)(2)?

The AIA does not provide a specific definition for the terms “directly” or “indirectly.” The phrase “by another who obtained the

subject matter disclosed directly or indirectly from the inventor or a joint inventor” is used throughout AIA 35 U.S.C. 102(b) to treat

a disclosure by a third party who obtained the subject matter disclosed from the inventor or a joint inventor (directly or indirectly) as

a disclosure of the subject matter by the inventor or a joint inventor. The phrase “directly or indirectly” is used to cover all situations

in which a third party obtained the subject matter disclosed from the inventor or a joint inventor, whether the subject matter was

communicated from the inventor or a joint inventor to the third party without an intermediary (directly) or was communicated from

the inventor or a joint inventor to the third party via an intermediary (indirectly).

Question FITF2920: Is it the applicant’s responsibility to state that the subject matter disclosed in the prior art was obtained from the

inventor of the application under examination to invoke the exceptions in 35 U.S.C. 102(b)(1) and (b)(2)?

If the publication, patent, or published application does not name the inventor or joint inventor as the author(s) or inventor or joint

inventor, the applicant must submit an affidavit or declaration under 37 CFR 1.130(a) establishing that subject matter disclosed

originated from the inventor or a joint inventor and that the subject matter was communicated by the inventor or a joint inventor,

directly or indirectly, to have the publication, patent, or published application treated as having been obtained from the inventor or a

joint inventor.

Page 25: First Inventor to File - Pass Patent Barpasspatentbar.com/wp-content/uploads/Quiz-4-First-Inventor-to-File.pdfFirst Inventor to File Effective Date of First-Inventor-to-File Provisions

Question FITF2925: Does the inventor need to show that he or she independently invented the subject matter to invoke the exceptions

in AIA 35 U.S.C. 102(b)(1) and (b)(2)?

The applicant must establish that the subject matter disclosed originated with the inventor or a joint inventor.

Question FITF2930: Is an affidavit by the inventor sufficient to prove that the disclosure was “by another who obtained the subject

matter directly or indirectly from the inventor or joint inventor” under the exception of AIA 35 U.S.C. 102(b)(2)(A)?

An affidavit or declaration under 37 CFR 1.130(a) or (b) is the correct mechanism for an applicant to show that a third party

(another) obtained the subject matter disclosed directly or indirectly from the inventor or a joint inventor. But whether a particular

affidavit or declaration under 37 CFR 1.130(a) or (b) is sufficient to establish that a third party obtained the subject matter disclosed

directly or indirectly from the inventor or a joint inventor is a case-by-case determination based upon the facts and evidence

presented in the affidavit or declaration.

Question FITF2935: Who decides whether a disclosure was obtained “indirectly from the inventor” for the exceptions in AIA 35

U.S.C. 102(b)(1) and (b)(2), and what is the measure for “indirectly from the inventor?”

The examiner would evaluate the inventor’s evidence submitted in an affidavit or declaration under 37 CFR 1.130 to determine

whether a disclosure of subject matter was “by another who obtained the subject matter disclosed directly or indirectly from the

inventor or a joint inventor.” It is not necessary for an examiner to determine whether the third party obtained the subject matter

disclosed directly from the inventor or a joint inventor or whether the third party obtained the subject matter disclosed indirectly

from the inventor or a joint inventor. The examiner need only determine whether the disclosure of subject matter was “by another

who obtained the subject matter disclosed directly or indirectly from the inventor or a joint inventor.”

Question FITF2940: What is an example situation where a third party may have “obtained the subject matter indirectly from the

inventor or joint inventor” for the exceptions in AIA 35 U.S.C. 102(b)(1) and (b)(2)?

An example of a disclosure of subject matter by another who obtained the subject matter disclosed indirectly from the inventor or a

joint inventor may occur when the inventor or a joint inventor communicates subject matter to an intermediary (party “X”), who in

turn communicates the subject matter to a third party (party “Y”), who discloses the subject matter. In this situation, there has been

a disclosure of subject matter by another (party “Y”) who obtained the subject matter disclosed indirectly from the inventor or a joint

inventor (via party “X”).

Question FITF2945: How can an inventor prove that an intervening third party disclosure under 35 U.S.C. 102(a)(1) is subject to the

exception of 35 U.S.C. 102(b)(1)(B) because it was obtained from the inventor and was not independently invented by the third party?

An inventor seeking to disqualify an earlier publication disclosed during the inventor’s grace period under the exception outlined in

35 U.S.C. 102(b)(1)(B) could submit an affidavit or declaration under 37 CFR 1.130(b). An affidavit or declaration under 37 CFR

1.130(b) alleging that the “subject matter” of the third party’s public disclosure is by another who obtained the subject matter

disclosed directly or indirectly from the inventor or a joint inventor must also establish that subject matter disclosed originated from

the inventor or a joint inventor and that the subject matter was communicated by the inventor or a joint inventor, directly or

indirectly, to the entity that made the disclosure of the subject matter. Any documentation which provides evidence of the

communication of the subject matter by the inventor or a joint inventor to the entity that made the disclosure of the subject matter

Page 26: First Inventor to File - Pass Patent Barpasspatentbar.com/wp-content/uploads/Quiz-4-First-Inventor-to-File.pdfFirst Inventor to File Effective Date of First-Inventor-to-File Provisions

should accompany the affidavit or declaration under 37 CFR 1.130(b). See Examination Guidelines for Implementing the first

Inventor to File Provisions of the Leahy-Smith America Invents Act, 78 Fed. Reg. 11059, 11081 (Feb. 14, 2013) (“Showing that the

disclosure was made, or that subject matter had been previously publicly disclosed, by another who obtained the subject matter

disclosed directly or indirectly from the inventor or a joint inventor”).

Question FITF2950: If an applicant is seeking to invoke the exceptions in AIA 35 U.S.C. 102(b)(1) and (b)(2), should the inventors use

an Information Disclosure Statement (IDS) to indicate that “the subject matter” in a third party’s public disclosure was obtained directly

or indirectly from the inventors?

No, a showing that the “subject matter” of the third party’s public disclosure is by another who obtained the subject matter disclosed

directly or indirectly from the inventor or a joint inventor would not be submitted in an IDS. The applicant should submit an

affidavit or declaration from an inventor under 37 CFR 1.130 to show that the subject matter in the disclosure was obtained directly

or indirectly from one of the inventor.

Question FITF2955: How does the exception in AIA 35 U.S.C. 102(b)(1)(B) operate since it appears that a third party is the only one

to know whether the invention was “obtained” from someone else or who was the source of their “disclosed subject matter?”

An affidavit or declaration under 37 CFR 1.130(b) alleging that the “subject matter” of the public disclosure is by another who

obtained the subject matter disclosed directly or indirectly from the inventor or a joint inventor must establish that (1) the subject

matter disclosed originated with the inventor or a joint inventor; and (2) the subject matter was communicated by the inventor or a

joint inventor, directly or indirectly. While AIA 35 U.S.C. 102(b)(1)(B) encompasses the situation in which the “another who

obtained the subject matter disclosed directly or indirectly from the inventor or a joint inventor” is adverse to the inventor or joint

inventor, it does not require that the “another” be adverse or a third party to the inventor or joint inventor. The “another” may be a

co-worker or collaborator of the inventor or joint inventor, or may be assignee-owner of the subject matter of the inventor or joint

inventor. Thus, an applicant may be able to provide a statement from the “another” in support of an affidavit or declaration under 37

CFR 1.130 that the subject matter of a public disclosure is by another who obtained the subject matter disclosed directly or indirectly

from the inventor or a joint inventor. Even if the “another” is adverse or a third-party to the inventor or joint inventor, an applicant

may still provide an affidavit or declaration under 37 CFR 1.130 establishing that subject matter disclosed originated with the

inventor or a joint inventor and that the subject matter was communicated by the inventor or a joint inventor, directly or indirectly.

Question FITF2960: How can an examiner determine that subject matter in a public disclosure by another was “obtained by the

inventor or joint inventor” for the 35 U.S.C. 102(b)(2) exceptions?

An examiner is not expected to determine that subject matter in a public disclosure by another (e.g., a publication, patent, or

published application naming someone other than the inventor or joint inventor as the author(s) or inventor or joint inventor) is

subject matter obtained from the inventor or a joint inventor in the absence of an appropriate affidavit or declaration under 37 CFR

1.130(a) establishing that subject matter disclosed originated from the inventor or a joint inventor.

Question FITF2965: How does the examiner determine the validity of an inventor’s statement that the disclosure was “by another who

obtained the subject matter directly or indirectly from the inventor or joint inventor?”

Page 27: First Inventor to File - Pass Patent Barpasspatentbar.com/wp-content/uploads/Quiz-4-First-Inventor-to-File.pdfFirst Inventor to File Effective Date of First-Inventor-to-File Provisions

The applicant would need to establish that the subject matter disclosed originated with the inventor or a joint inventor and that the

subject matter was communicated by the inventor or a joint inventor, directly or indirectly. Any documentation which provides

evidence of the communication of the subject matter by the inventor or a joint inventor to the entity that made the disclosure of the

subject matter should accompany the affidavit or declaration. See Examination Guidelines for Implementing the First Inventor To

File Provisions of the Leahy-Smith America Invents Act, 78 Fed. Reg. 11059, 11081 (Feb. 14, 2013) (“Showing that the disclosure

was made, or that subject matter had been previously publicly disclosed, by another who obtained the subject matter disclosed

directly or indirectly from the inventor or a joint inventor“).

Back to top

USPTO Electronic System Indications of First-Inventor-to-File Applications

Question FITF3000: How will an applicant know whether his/her application will be examined under the first-to-invent provisions of

pre-AIA law or the first-inventor-to-file provisions of the AIA?

An applicant will be notified of the statutory framework to be applied to his/her application in a variety of ways.

First, there will be a viewable field in the application contents tab of PRIVATE PAIR (for unpublished applications) and a similar

field in PUBLIC PAIR (for published applications and patents) indicating either “AIA (First Inventor To File): yes” for an

application to be examined under the first-inventor-to-file provisions or “AIA (First Inventor To File): no” for an application to be

examined under the first-to-invent provisions. The field will show a hyphen for applications not yet ready for examination.

Second, certain form paragraphs used for Office actions will state whether the application is examined under the first-to-invent

provisions of pre-AIA law or the first-inventor-to-file provisions of the AIA.

Third, certain USPTO forms that accompany Office actions will indicate “AIA (First Inventor to File) Status: Yes” for an

application to be examined under the first-inventor-to-file provisions or “AIA (First Inventor to File) Status: No” for an application

to be examined under the first-to-invent provisions.

Back to top

Statement Under 37 CFR 1.55 or 1.78

Question FITF3110: If an applicant is required to submit a statement under 37 CFR 1.55 or 1.78 to indicate that an application filed on

or after March 16, 2013 with a domestic benefit or foreign priority claim before March 16, 2013, contains claim(s) lacking support in the

application filed before March 16, 2013 (i.e., should be examined under the first-inventor-to-file provision), what is an example of a

compliant statement?

Here are four examples of a compliant 1.55/1.78 statement that an applicant may file with the Office:

o This application filed on or after March 16, 2013 which claims priority to an application filed before March 16, 2013, contains one or

more claims NOT entitled to a filing date before March 16, 2013.

o This application claims priority to an application filed before March 16, 2013, and contains one or more claims NOT entitled to a filing

date before March 16, 2013.

o This application contains one or more claims NOT entitled to a filing date before March 16, 2013.

Page 28: First Inventor to File - Pass Patent Barpasspatentbar.com/wp-content/uploads/Quiz-4-First-Inventor-to-File.pdfFirst Inventor to File Effective Date of First-Inventor-to-File Provisions

o This application contains a claim having an effective filing date on or after March 16, 2013.

Question FITF3120: If an applicant intends to file a 37 CFR 1.55 or 1.78 statement indicating that the application contains a claim that

is supported only by an application filed on or after March 16, 2013, where should the applicant place the statement?

Updated versions of the Application Data Sheet (ADS, Form PTO/AIA/14) and the 371 Transmittal Letter (Form PTO-1390), each

with a checkbox, are available for the applicant to make the statement on filing. For follow-on papers, the applicant may provide the

statement or rescind such a statement in a separate document. Applicants must select the document description “Make/Rescind AIA

First Inventor to File) 1.55/1.78 Stmnt” to index this separate document when filing online.

Question FITF3130: If a provisional application is filed before March 16, 2013, is a 37 CFR 1.55 or 1.78 statement required in the

nonprovisional application filed on or after March 16, 2013?

As long as each claim ever presented in the nonprovisional application is fully supported under 35 U.S.C. 112(a) by the disclosure of

the provisional application, the nonprovisional application is a pre-AIA (FTI) application, and no 1.55/1.78 statement is required.

Question FITF3140: For transition applications, if the claims are amended, but the amended claims are fully supported by a foreign

priority or domestic benefit application filed before March 16, 2013, is a 37 CFR 1.55 or1.78 statement required? If an amended claim

has slightly different wording than the claims of the priority application, but it is not considered new matter, is a statement required?

No. An applicant is only required to provide a statement if an application filed on or after March 16, 2013, claims priority to or the

benefit of an application filed prior to March 16, 2013, and contains or ever contained a claim to a claimed invention having an

effective filing date as defined in 35 U.S.C. 100(i) that is on or after March 16, 2013. Changes in wording that do not amount to new

matter do not affect the effective filing date of the claim and therefore a 1.55/1.78 statement is not required.

Question FITF3150: If an application containing broad claims has been allowed and the applicant files a continuation-in-part

application disclosing and claiming a new narrow embodiment that is covered by the broad claims of the allowed parent application but

which is not specifically described in the allowed parent application, does the applicant need to provide a 37 CFR 1.55 or 1.78 statement

and what is effective file date for these very narrow claims?

If the “very narrow claims” in the continuation-in-part application are not supported under 35 U.S.C. 112(a) by the disclosure of the

prior pre-AIA (FTI) application, the effective filing date of these claims is the actual filing date of the continuation-in-part

application. An applicant is required to provide a statement if an application filed on or after March 16, 2013, claims priority to or

the benefit of an application filed prior to March 16, 2013, and contains or ever contained a claim to a claimed invention having an

effective filing date on or after March 16, 2013. A 1.55/1.78 statement would be required in this situation because the continuation-

in-part application contains a claim to a claimed invention having an effective filing date on or after March 16, 2013.

Question FITF3160: If a 37 CFR 1.55 or 1.78 statement is filed in an application, does a statement also have to be filed in response to

every Office action submitted thereafter, and in every later-filed continuation or divisional application?

No. An applicant is required to file a 1.55/1.78 statement only once in an application or in a parent application. A 1.55/1.78

statement filed in an application automatically carries over to the continuation or divisional application.

Question FITF3170: If an applicant files a 37 CFR 1.55 or 1.78 statement, how will the USPTO acknowledge the statement?

Page 29: First Inventor to File - Pass Patent Barpasspatentbar.com/wp-content/uploads/Quiz-4-First-Inventor-to-File.pdfFirst Inventor to File Effective Date of First-Inventor-to-File Provisions

If an applicant uses the updated Application Data Sheet (ADS, Form PTO/AIA/14) which provides a checkbox so that an applicant

can make the statement, the Office of Patent Application Processing (OPAP) will indicate on the filing receipt that such a statement

was made. Alternatively, if an applicant uses the updated 371 Transmittal Letter (Form PTO-1390) which provides a checkbox so

that an applicant can make the statement, OPAP will indicate on the filing receipt that such a statement was made. Lastly, if an

applicant makes the statement in a separate paper during the course of prosecution, the Technology Center staff will indicate on a

corrected filing receipt that such a statement was made.

Question FITF3180: Can an applicant file a late 37 CFR 1.55 or 1.78 statement in a transition application, for example because the

applicant unintentionally overlooked the need to file the statement?

An applicant should file a 1.55/1.78 statement as soon as the applicant becomes aware that the statement was needed and

unintentionally overlooked in order to make sure that the application will be examined under the correct statutory framework. For

example, if the application has been allowed and the issue fee paid before applicant files the 1.55/1.78 statement, the applicant may

have to withdraw the application from issue and file an RCE in order for the 1.55/1.78 statement to be considered, and the

application examined under the correct statutory framework.

Question FITF3190: If an applicant erroneously files a 37 CFR 1.55 or 1.78 statement indicating that an application filed on or after

March 16, 2013 with a domestic benefit or foreign priority claim before March 16, 2013, contains one or more claims lacking support in

the application filed before March 16, 2013 (i.e., should be examined under the first-inventor-to-file provisions), how should an applicant

correct the error?

If an applicant erroneously submits a statement under 37 CFR 1.55 or 1.78, the applicant should file a separate paper to rescind the

statement. If the applicant files the separate paper of rescission electronically, the applicant should select the document description

"Make/Rescind AIA (First Inventor to File) 1.55/1.78 Stmnt " to properly index the paper.

Back to top

Interim Copies of Foreign Priority Documents

Question FITF3300: If an applicant submits an interim copy of a foreign priority application, will the applicant still need to provide a

certified copy of the foreign priority document? How will an interim copy submission be reflected in the USPTO’s records?

Yes. If an applicant files an interim copy of a foreign priority application, the applicant must provide a certified copy of the foreign

priority document before patent grant in order to perfect the priority claim. An interim copy of a foreign priority application will be

entered into the file wrapper of the application in which it was filed and labeled as “Interim Copy of the Foreign Priority Document.”

Question FITF3310: Are there any special instructions for filing an interim copy of the foreign priority application?

Yes. Main parts of the interim copy of the foreign priority application, i.e., Abstract, Drawings, Specification, Claims, and possibly

Sequence Listing or Computer Program Listings, along with the separate cover sheet required by 37 CFR 1.55(i), may be filed as a

single PDF file via EFS-Web (or as a single paper document). The PDF file will be stored in the Image File Wrapper of the

application, and applicants must select the document description “Interim Copy of the Foreign Priority Application” to index this

document when filing online.

Page 30: First Inventor to File - Pass Patent Barpasspatentbar.com/wp-content/uploads/Quiz-4-First-Inventor-to-File.pdfFirst Inventor to File Effective Date of First-Inventor-to-File Provisions

Application parts that were filed in a foreign patent office only as plain text computer files (e.g. TXT file), such as a Sequence

Listing, lengthy Tables(s), or Computer Program Listing appendix, may be filed as a TXT file via EFS-Web (or on compact

disc). When filing online via EFS-Web, applicants must select the document description “Interim Copy of the Foreign Priority (text

file)” to index a plain text file part of the application. Please note that EFS-Web will not perform preliminary validation on an

interim sequence listing text file, and that an interim sequence listing over 25 MB must be split into multiple files under the 25 MB

upload limit. If a TXT file is filed on compact disc, the compact disc must be labeled as “Text File of Interim Copy of Foreign

Priority Application” along with (1) the country code and application number of the foreign priority document, (2) at least two

pieces of identifying information for the application (e.g., application number, docket number, title, etc.) in which the interim copy

of the foreign application was filed, and (3) if multiple compact discs are submitted, a label indicating their order (e.g. “1 of X”).

The file name of each TXT file must begin with the country code and application number of the foreign priority application of which

it is a part (any punctuation or space in the application number must either be omitted or replaced with a dash “-“ or underscore ”_”

character); e.g. “EP111838637_SequenceListing.txt” or “EP_11183863_7_Table 2”. Where an original TXT file must be split into

multiple parts, the file name for each part should indicate the order; for example, EP111838637_SeqList_part1of2.txt and

EP111838637_SeqList_part2of2.txt.

Back to top

Foreign Filing Date and Translations

Question FITF3410: Does 35 U.S.C. 112(a) need to be satisfied in the previously-filed domestic benefit or foreign priority application

in order for a claimed invention under examination to be entitled to rely on the filing date of the previously-filed application to overcome

a rejection?

Yes. In order for a claimed invention to be entitled to rely on the filing date of a previously-filed domestic benefit or foreign priority

application to overcome a rejection, 35 U.S.C. 112(a) must be satisfied in the previously-filed document. The standard for

entitlement to the filing date of a previously filed domestic or foreign application has not changed. The domestic benefit or foreign

priority application must support the invention claimed for the applicant to obtain the benefit of or priority to the filing date of the

previously filed domestic benefit or foreign priority application.

Question FITF3420: Does an applicant need to claim priority to a prior-filed foreign application to get the benefit of the filing date of

the foreign application?

Yes. An applicant must file a claim for foreign priority within four months of the actual filing date of the application or within

sixteen months from the filing date of the prior foreign application.

Question FITF3430: Does an applicant need to provide a translation of a foreign priority application to: (1) claim priority to the

foreign application; (2) determine the effective filing date of the claims for purposes of determining whether the application is an AIA

(FITF) application or a pre-AIA (FTI) application; (3) ensure that the subject matter of the U.S. nonprovisional application is supported

by the foreign priority application; or (4) disqualify a reference with a prior effective date?

The applicant does not need to provide an English-language translation of a foreign application to claim priority to the foreign

application or to determine the effective filing date of the claims for purposes of determining whether the application is an AIA

(FITF) application or a pre-AIA (FTI) application. The applicant, however, will be required to provide a certified copy of the

Page 31: First Inventor to File - Pass Patent Barpasspatentbar.com/wp-content/uploads/Quiz-4-First-Inventor-to-File.pdfFirst Inventor to File Effective Date of First-Inventor-to-File Provisions

foreign priority application and an English-language translation (if in a language other than English) if the filing date of the foreign

priority application is necessary to overcome a rejection.

Question FITF3450: Is an applicant entitled to receive the benefit of a previously filed foreign priority application if the foreign

priority document is not in the English language?

Yes, an applicant is entitled to the benefit of a previously-filed, non-English foreign priority application, but the applicant will be

required to provide an English-language translation of the foreign application if the filing date of the foreign application is necessary

to overcome a rejection.

Question FITF3460: In determining whether the claimed invention of an application is entitled to a foreign priority date, does the

examiner verify that the subject matter of the claims in the application is supported in the foreign priority document?

Yes. In order to decide whether the effective filing date of the claimed invention is the filing date of the foreign priority application,

the examiner checks to see whether the claimed invention is supported in the foreign priority document.

Question FITF3470: How would an examiner verify that subject matter disclosed in a published application is supported by the foreign

priority application?

Examiners have various ways to verify that subject matter disclosed in the published application is supported by a foreign language

priority application. Examiners have access to desktop electronic translation tools. Examiners also may consult USPTO

translators. Lastly, the foreign language priority patent application may have an English language patent family member.

Back to top

Foreign Priority Documents

Question FITF3500: At the time of filing, an applicant uses an Application Data Sheet (ADS, Form PTO/AIA/14) to make a timely

foreign priority claim to an application filed in a foreign intellectual property office that does not participate in the priority document

exchange program. However, within 4 months of the actual filing date or 16 months from the filing date of the priority document, the

applicant fails to submit either a certified copy of the foreign priority document (a certified copy was not filed in a parent or related

application), form PTO/SB/38 to request retrieval of the priority document under 37 CFR 1.55(h)(4), or an interim copy of the foreign

priority document under 37 CFR 1.55(i). What process and fees are required for submitting these documents after the 4/16 month time

period?

If an applicant misses the 4/16 month deadline as set forth in 37 CFR 1.55 for filing either the certified copy, form PTO/SB/38, or an

interim copy, but timely made the foreign priority claim on the ADS, the applicant can submit any one of these documents, where

applicable, with a petition under 37 CFR 1.55(f). If the applicant files a separate written request form PTO/SB/38 to request

retrieval of the priority document contained in an application filed with a participating foreign intellectual property office or an

interim copy with the petition under 37 CFR 1.55(f), the electronic copy of the priority document must be received or the certified

copy must be filed before patent grant in order to be entitled to the right of priority. The petition under 37 CFR 1.55(f) must include

a showing of good and sufficient cause for the delay and the petition fee set forth in 37 CFR 1.17(g). A mere statement that the

delay was unintentional or that there was good and sufficient cause for the delay is insufficient. The 37 CFR 1.55(f) showing

requires a brief explanation as to the good and sufficient cause for the delay. If the applicant identifies the parent or related

application that contains the certified copy, a petition under 37 CFR 1.55(f) is not required.

Page 32: First Inventor to File - Pass Patent Barpasspatentbar.com/wp-content/uploads/Quiz-4-First-Inventor-to-File.pdfFirst Inventor to File Effective Date of First-Inventor-to-File Provisions

Back to top

Obviousness

Question FITF8010: Are there still obviousness rejections under the AIA?

Yes, the AIA amended 35 U.S.C. 103 to provide that a claimed invention is not patentable if the differences between the claimed

invention and the prior art are such that the claimed invention as a whole would have been obvious before the effective filing date of

the claimed invention to a person having ordinary skill in the art to which the claimed invention pertains. This provision tracks pre-

AIA 35 U.S.C. 103(a) in substance, except that the temporal focus for the obviousness inquiry under the AIA is before the effective

filing date of the claimed invention, rather than at the time of the invention as applicable under pre-AIA law.

Question FITF8020: How is 35 U.S.C. 103 affected under the AIA?

AIA 35 U.S.C. 103 tracks pre-AIA 35 U.S.C. 103(a), except that the temporal focus for the obviousness inquiry is before the

effective filing date of the claimed invention, rather than at the time of the invention. The provisions of pre-AIA 35 U.S.C. 103(c)

have been replaced with 35 U.S.C. 102(b)(2)(C) and (c), and the provisions of pre-AIA 35 U.S.C. 103(b) pertaining to

biotechnological processes have been eliminated.

Back to top

Miscellaneous

Question FITF9010: What are the differences between an inventor, a joint inventor, and an inventive entity?

The AIA defines the term “the inventor” as the individual or, if a joint invention, the individuals collectively who invented or

discovered the subject matter of the invention. The terms “inventorship” and “inventive entity” are synonymous with the term “the

inventor” as defined by the AIA. The AIA defines the terms “joint inventor” and “co-inventor” as any one of the individuals who

invented or discovered the subject matter of a joint invention.

Question FITF9020: Does the AIA define a “third party” disclosure?

No, the AIA does not use or define the term “third party” with respect to the first-inventor-to-file provisions of 35 U.S.C. 102 and

103. A “third party” disclosure as used in the Examination Guidelines and training materials means a disclosure that names a person

or persons not named as the inventor or a joint inventor for the application under examination.

Question FITF9030: Does the AIA change how patent term adjustment is calculated?

No, the AIA does not change the patent term adjustment provisions of 35 U.S.C. 154(b) for applications filed under 35 U.S.C.

111(a). The AIA Technical Corrections Bill changes the patent term adjustment provisions of 35 U.S.C. 154(b) with respect to PCT

applications entering the national stage under 35 U.S.C. 371. The fourteen-month time frame in 35 U.S.C. 154(b)(1)(A)(i) and the

three-year time frame in 35 U.S.C. 154(b)(1)(B) are measured from the actual filing date of the application for applications filed

under 35 U.S.C. 111(a) and from the date the national stage commenced under 35 U.S.C. 371 in PCT applications entering the

national stage. The effective filing date of a claimed invention is not used in determining patent term adjustment under 35 U.S.C.

154(b).

Question FITF9040: Does the AIA change the way PCT applications are reviewed by the USPTO as an International Search Receiving

Office?

Page 33: First Inventor to File - Pass Patent Barpasspatentbar.com/wp-content/uploads/Quiz-4-First-Inventor-to-File.pdfFirst Inventor to File Effective Date of First-Inventor-to-File Provisions

No. The USPTO will continue to follow the PCT rules, including PCT Rule 33.1 concerning search of the prior art, when acting as

an International Search Receiving Office.

Question FITF9050: How is double patenting affected by AIA?

The AIA does not change double patenting.